Transcript

FÍSICA – MECÂNICA

AUTORIA – PROF. MARCELO CORREIA E-mail: [email protected] 1

+++++++++++++++++++++++++++++++++++++++++++++++++++++++++++++++++++++++++++++++++++++++ Nesta parte estudaremos a DINÂMICA – parte da Mecânica que estuda os movimentos preocupando-se com as causas que os provocam ou os alteram.

Constata-se que em todas as provas de vestibulares é dada uma grande ênfase a Mecânica, portanto é muito importante o entendimento detalhado desta parte da Física e além do mais a Mecânica constituirá uma base indispensável para o entendimento de outros ramos da Física. Durante o passar dos anos

compreendeu-se que a forma de passar no vestibular é muito simples, requer dedicação e uma fórmula mágica: estudar, estudar, estudar, ... O professor de Física, MARCELO CORREIA.

DINÂMICA – PARTE 1 A dinâmica é a parte da Mecânica que estuda os movimentos levando em consideração as causas que os provocam ou os alteram.

LEIS DE NEWTON As Leis de Newton constitui a base da Mecânica Clássica e são três:

1ª Lei de Newton – Princípio da Inércia O Princípio da inércia pode ser enunciado de diversas formas, então apresentaremos abaixo algumas formas de enunciar a 1ª Lei de Newton:

TODO CORPO TEM A TENDÊNCIA DE PERMANECER NO SEU ESTADO DE REPOUSO OU MRU AO MENOS QUE UM AGENTE

EXTERNO ATUE SOBRE ELE.

TODO CORPO PERMANECERÁ EM REPOUSO OU EM MRU, INDEFINIDAMENTE AO MENOS QUE UM AGENTE EXTERNO ATUE

SOBRE ELE.

QUALQUER VELOCIDADE, UMA VEZ TRANSMITIDA A UM CORPO, SERÁ MANTIDA INDEFINIDAMENTE, DESDE QUE SOBRE O CORPO

NÃO ATUE CAUSAS PARA AQUISIÇÃO DE ACELERAÇÃO. Do Princípio da Inércia observamos que existem dois estados naturais para o corpo, isto é, existem dois estados em que não é necessária a atuação de agente externo para que o corpo esteja neles. Estes estados são: REPOUSO E MRU (Movimento Retilíneo e Uniforme). Assim, podemos dizer que quando um corpo se encontra em repouso ou em MRU ele está em equilíbrio. Portanto temos dois tipos de equilíbrio:

� Repouso ���� Equilíbrio Estático; � MRU ���� Equilíbrio Dinâmico.

2ª Lei de Newton – Princípio Fundamental da Dinâmica

Observando que para um corpo não estar em repouso ou em MRU deve sobre ele atuar um agente externo Newton definiu uma grandeza física que caracteriza quantitativamente a ação do agente externo e a esta grandeza deu o nome de força. Assim, a 2ª Lei de Newton pode ser enunciada da forma seguinte:

A FORÇA RESULTANTE QUE ATUA SOBRE UMA PARTÍCULA É DIRETAMENTE PROPORCIONAL AO PRODUTO DE SUA MASSA (m) PELA

ACELERAÇÃO ADQUIRIDA POR ELA ( ar).

Escrevendo o Princípio Fundamental da Dinâmica matematicamente

temos: Onde:

RFr

���� força resultante

m ���� massa do corpo amFR

rr

⋅=

ar

���� aceleração

Devemos atentar para o seguinte: � A massa de um corpo é uma medida de sua inércia. Quanto maior a

massa de um corpo maior a sua inércia, isto é, maior a sua resistência a alterações do seu estado de repouso ou MRU;

� Sabemos que um corpo em equilíbrio estático ou dinâmico não atua

agente externo, isto é, 0FR =r

. Portanto quanto um corpo está em

repouso ou em MRU a força resultante que a tua sobre ele é nula, isto é o que chamamos de condição de equilíbrio de translação para uma partícula.

� FORÇA RESULTANTE é a soma vetorial de todas as forças que atua no corpo.

� A unidade de força no SI é: kg� m/s2 que recebe o nome especial de newton – N.

� 1N é a força para que um corpo de 1kg adquira uma aceleração de 1m/s2.

3ª Lei de Newton – Princípio da Ação e Reação

O princípio da ação e reação pode ser enunciado da forma seguinte:

QUANTO UM CORPO “A” APLICA UMA FORÇA Fr

NUM CORPO “B” O

CORPO “B” REAGE E APLICA NO CORPO “A” UMA FORÇA Fr

− QUE TEM MESMO MÓDULO, MESMA DIREÇÃO E SENTIDO OPOSTO.

FORÇAS DE GRANDE IMPORTÂNCIA

Força Peso Pr

A força peso é a força com que a Terra (ou outro corpo celeste massivo) atrai os corpos que estão em suas proximidades para o seu centro.

Observado de Fora da Terra Observado da Terra

No nosso caso, resolveremos a maioria das situações observando o fenômeno aqui da Terra, sendo assim a força peso que atua em uma partícula será uma força com direção vertical e sentido apontando para baixo. Sendo o peso uma força deve obedecer a 2ª Lei de Newton, assim temos:

gmPrr

⋅=

gr

���� aceleração da gravidade. Nas

proximidades da terra g = 100m/s2.

da Universidade de Pernambuco

Pré- Vestibular

DINÂMICA DA PARTÍCULA – Força, Energia, Momento Linear e Impulso. SISTEMA DE PARTÍCULAS e CORPO RÍGIDO – Centro de Massa, Rotação, Torque, Momeno Angular. ESTÁTICA – Estática do ponto material, do corpo extenso e Fluidostática.

AUTORIA

MARCELO CORREIA

Pr

m

Pr

Terra

corpo

Força peso

Reação da Força Peso

Pr

m

Superfície terrestre

FÍSICA – MECÂNICA

AUTORIA – PROF. MARCELO CORREIA E-mail: [email protected] 2

Reação Normal ou Normal Nr

A normal é uma força de reação provocada por um apoio, ou seja, é a força aplicada a um corpo pela superfície em que está apoiado. Não esqueça que:

� A normal não é a reação da força peso; � A ação que provoca a normal é aplicada no apoio, isto é, no corpo

em que o corpo está apoiado; � Não há uma fórmula pronta para calcular a normal, portanto

devemos aplicar as Leis de Newton para encontrar a normal; � Podemos, de forma coloquial, dizer que a normal é uma medida

para o quanto sofre o apoio. Vejamos algumas situações:

Tração ou Tensão Tr

Chamamos de tração ou tensão a força que é transmitida através de um fio ou algo semelhante. Fio ideal é aquele que não tem massa, portanto não tem inércia, e é inextensível (não se deforma). Não esqueça que:

� Para um mesmo fio ideal a tração nos seus extremos tem mesmo módulo e sentidos opostos.

Veja algumas situações abaixo:

Força Elástica – Lei de Hooke A Lei de Hooke trata da força elástica e pode ser enunciada da seguinte forma: AO APLICARMOS UMA FORÇA A UMA MOLA (CORPO ELÁSTICO) EM SEU

REGIME ELÁSTICO O MÓDULO DA FORÇA EXERCIDA PELO CORPO ELÁSTICO PARA RETORNAR AO SEU ESTADO NATURAL (LIVRE DE FORÇAS) É DIRETAMENTE PROPORCIONAL A SUA DEFORMAÇÃO.

A lei de Hooke escrita matematicamente é:

Fel ���� Força elástica K ���� Constante elástica (depende da mola) xKFel ⋅=

X ���� Deformação

Veja abaixo:

Nota: Um corpo está no regime elástico quando após a retirada das forças que o deformam ele retorna ao seu estado inicial.

Força de Atrito atFr

A força de atrito é uma força de resistência a tendência do movimento. A força de atrito tem origem em forças eletromagnéticas e por conta das irregularidades das superfícies que tendem a escorar entre si. Consideramos dois tipos de forças de atrito:

� Força de atrito dinâmica ou cinética ���� é a força de atrito que atua sobre o corpo quando este está em movimento (v ≠ 0).

A experiência mostra que a intensidade da força de atrito dinâmica é diretamente proporcional a reação do apoio, isto é, é diretamente proporcional a

normal Nr

. Assim teremos: Fat ���� Força atrito µd ���� Coeficiente de atrito dinâmico NF dat ⋅= µ N ���� Reação do apoio – Normal

� Força de atrito estática ���� é a força de atrito que atua sobre o corpo quando este está em repouso (v = 0).

A experiência mostra que a intensidade da força de atrito estática

máxima é diretamente proporcional a normal Nr

. Fat ���� Força atrito µe ���� Coeficiente de atrito estático NF eat(máx) ⋅= µ N ���� Reação do apoio – Normal

É muito importante perceber que a expressão anterior nos fornece a força de atrito estática máxima e esta não é necessariamente a força de atrito estática que pode estar atuando no corpo. Observamos que a força de atrito estática pode ter valores compreendidos entre zero e o valor máximo dependendo da força que solicita a movimentação do corpo (que permanece em

repouso). Assim podemos escrever: at(máx)at FF0 ≤≤

As forças de atrito estática e dinâmica não são iguais observe que há dois coeficientes de atrito: o dinâmico (µd) e o estático (µe). Sabemos que o coeficiente de atrito estático é ligeiramente maior que o coeficiente de atrito dinâmico, assim a força de atrito estática máxima é maior que a força de atrito dinâmica. Este é o motivo pelo qual é mais fácil manter um movimento do que iniciar um movimento. É muito importante perceber também que a força de atrito não tem qualquer dependência com a área de contato entre as superfícies.

Nr

Teto

Nr

θ

Plano Inclinado

Nr

Parede Nr

Mesa

A Fr

Tr

Tr

− B

Pr

Tr

Tr

A

B

Fr

elFr

xf

xi x

Mola livre de forças. Estado

natural.

Mola deformada. Aparece a força

elástica.

FÍSICA – MECÂNICA

AUTORIA – PROF. MARCELO CORREIA E-mail: [email protected] 3

Força de Resistência dos Fluidos Rr

Quando um corpo se movimenta imerso em um fluido (liquido ou gás); como o ar, por exemplo; aparece uma força de atrito causada pelo fluido ou força de atrito viscoso ou simplesmente força de resistência. No caso do ar dizemos que é a força de resistência do ar. Observamos que experimentalmente a força

de resistência Rr

tem módulo diretamente proporcional a potência n do modulo da velocidade do fluido. Assim, podemos escrever:

nvkR ⋅= onde:

� R ���� Intensidade da força de resistência; � n ���� Constante que depende da ordem de grandeza da velocidade e

do tamanho do corpo. Para maioria dos casos: n = 1 ou n = 2; � K ���� Constante que depende da natureza do fluido (densidade,

temperatura) e depende da maior área de contado do corpo com o fluido perpendicularmente a direção do movimento.

Nunca esqueça que a força de resistência sempre tem sentido contrário ao movimento do corpo.

PLANO INCLINADO O plano inclinado é um dispositivo que aparece muito e, portanto vamos mostrar uma análise básica para se resolver problemas envolvendo o plano inclinado. No caso que vamos mostrar não há atrito, no entanto se houver é só incluir esta mais esta força.

sen θPPt ⋅=

cos θPPN ⋅=

FORÇA EM TRAJETÓRIA CURVILÍNEA – FORÇA CENTRÍPETA CPFr

Sabemos da cinemática vetorial que uma partícula descrendo trajetória curvilínea é acelerada. Se uma partícula está se deslocando numa trajetória circular há a aceleração centrípeta que indica variação na direção da velocidade vetorial da partícula. Agora que já estudamos a 2ª Lei de Newton sabemos que toda alteração de movimento (alteração de velocidade – aceleração) é causada por uma força. A força centrípeta é a força que provoca a aceleração centrípeta de uma partícula e, portanto provoca variação na direção da velocidade vetorial da partícula. Aplicando a 2ª Lei de Newton para a aceleração centrípeta temos:

CPCP amFrr

⋅=

Lembrando que: R

va

2

CP = ou Rωa 2CP ⋅= , podemos calcular o

módulo da força centrípeta por:

2

vmF

2

CP ⋅= ou RωmF 2CP ⋅⋅=

onde: � V ���� Módulo da velocidade vetorial da partícula; � ω ���� velocidade angular da partícula; � R ���� Raio da trajetória descrita pela partícula.

Não esqueça que a força centrípeta tem mesma direção e mesmo sentido da aceleração centrípeta, isto é, aponta para o centro da trajetória.

BRINQUEDINHO DE VESTIBULANDO!!!!!

1. (UAAM) Um pescador está sentado sobre o banco de uma canoa. A Terra aplica-lhe uma força de atração gravitacional chamada peso. De acordo com a 3ª Lei de Newton, a reação dessa força atua sobre:

(a) a canoa. (b) o banco da canoa. (c) a água. (d) a Terra. (e) a canoa e a água e depende de canoa estar em repouso ou em movimento

2. (PUC–SP) No arremesso de peso, um atleta gira um corpo rapidamente e depois o abandona. Se não houvesse a influência da Terra, a trajetória do corpo após ser abandonado pelo atleta seria:

(a) Circular. (b) Parabólica. (c) Curva qualquer. (d) Retilínea. (e) Espira. (f)

3. Uma partícula sob a ação de várias forças cuja resultante é zero. Podemos afirmar que a partícula:

(a) Está em pouso. (b) Está em movimento acelerado. (c) Está em movimento circular. (d) Está em movimento retilíneo uniforme. (e) Pode estar em repouso ou em movimento retilíneo uniforme.

4. (FUVEST–SP) Um veículo de massa 5,0 kg descreve uma trajetória retilínea e obedece à equação horária: S = 3� t2 + 3� t + 1, onde S é medido em metros e t em segundos. Qual o módulo da força resultante sobre o veículo?

5. (Fund. Carlos Chagas–SP) Para que um carrinho de massa m adquira uma certa aceleração de módulo a é necessário que a força resultante tenha módulo F. Qual é o módulo da força resultante para que um carrinho de massa 2� m adquira uma aceleração de módulo 3� a?

(a) 1,5F (b) 2F (c) 3F (d) 5F (e) 6F

6. (FE Itajubá–MG) Um corpo cujo peso é 4,0 N, sob a ação de uma força constante, horizontal, de valor 3,0 N, descreve uma trajetória retilínea sobre uma mesa horizontal, com uma velocidade constante de 2,0 m/s. Quanto vale o módulo da resultante das forças que atuam sobre o corpo?

(a) 0,0N (b) 3,0N (c) 5,0 (d) Não se pode dizer coisa alguma a respeito do valor da resultante, uma vez

que a situação descrita no problema é fisicamente impossível. (e) Não se pode dizer coisa alguma a respeito do valor da resultante, uma vez

que, para isso, o problema não apresenta dados suficientes.

7. (ITA–SP) Em seu livro “Viagem ao Céu”, Monteiro Lobato, pela boca de um personagem, faz a seguinte afirmação: “Quando jogamos uma laranja para cima, ela sobe enquanto a força que produziu o movimento é maior do que a força da gravidade. Quando esta se torna maior, a laranja cai”. (Despreze a resistência do ar)

(a) A afirmação é correta pois, de F = m� a, temos que a = 0 quando F = 0. indicando que as duas forças se equilibram no ponto mais alto da trajetória.

(b) A afirmação está errada porque a força exercida para elevar a laranja sendo constante nunca será menor que a da gravidade.

(c) A afirmação está errada porque, após ser abandonada no espaço, a única força que age sobre a laranja é a da gravidade.

(d) A afirmação está correta porque está de acordo com o Princípio da Ação e Reação.

(e) Não podemos tirar qualquer conclusão sobre a afirmação.

8. (AEU–DF) As Leis de Newton da Dinâmica são verificadas: (a) Só para observadores em repouso. (b) Para quaisquer observadores. (c) Só para observadores em movimento acelerado. (d) Para observadores parados ou com aceleração vetorial nula em relação a

um sistema inercial. (e) Só para observadores em movimento uniforme.

9. (PUC–SP) O sistema representado no desenho, de massa total 100 kg, é

puxado para a direita por uma força Fr

que o acelera uniformemente sobre trilhos sem atrito. O dinamômetro D ligado à esfera E, de massa 10 kg, que

Pr

Nr

x

NPr

xt PPrr

=

y

θ

θ

FÍSICA – MECÂNICA

AUTORIA – PROF. MARCELO CORREIA E-mail: [email protected] 4

pode deslizar sem atrito sobre a prancha horizontal, acusa uma força de 5 N

durante a aceleração. A aceleração que Fr

comunica ao sistema: (a) Não pode ser determinado. (b) Vale 0,05m/s2. (c) Vale 0,5m/s2.

(d) Vale 510

5m/s2.

(e) Vale 90

5m/s2.

10. (ITA–SP) A velocidade de uma partícula, num determinado instante t, é nula em relação a um referencial inercial. Pode-se afirmar que o no instante t:

(a) A resultante das forças que agem sobre a partícula é necessariamente nula. (b) A partícula se encontra em repouso, em relação a qualquer referencial

inercial. (c) A resultante das forças que agem sobre a partícula pode não ser nula. (d) A resultante das forças que agem sobre a partícula não pode ser nula. (e) Nenhuma das anteriores é verdadeira.

11. (Mackenzie–SP) Um elevador começa a subir, a partir do andar térreo, com aceleração de 5 m/s2. O peso aparente de um homem de 60 kg no interior do elevador, supondo g = 10 m/s2, é igual a:

(a) 60N (b) 200N (c) 300N (d) 600N (e) 900N

12. (ITA–SP) No teto de um elevador temos um corpo de peso 16 N preso a um dinamômetro que acusa 20 N. A aceleração local da gravidade vale 10 m/s2. A intensidade da aceleração do elevador é:

(a) zero (b) 2,5m/s2 (c) 5,0m/s2 (d) 10,0m/s2 (e) n.d.a.

13. (ITA–SP) Em relação à situação da questão anterior, podemos afirmar que o elevador está:

(a) subindo com velocidade constante. (b) Em repouso. (c) Subindo em movimento acelerado. (d) Descendo em movimento acelerado. (e) Subindo em movimento acelerado ou descendo em movimento retardado. 14. (ITA–SP) No sistema esquematizado são desprezíveis o atrito, o momento

de inércia da roldana e a massa do fio que liga as massas m1 e m2. Sabe-se que m1 > m2 e que a aceleração da gravidade local é g. A tensão T no fio e a aceleração a da massa m1 são, respectivamente, dadas por:

(a) 21

21

mm

gmm2T

+

⋅⋅⋅= ;

( )21

21

mm

gmma

+

⋅−=

(b) 21

21

mm

gmmT

+

⋅⋅= ;

( )21

21

mm

gmma

+

⋅−=

(c) ( ) gmmT 21 ⋅−= ; ( )

21

21

mm

gmma

+

⋅−=

(d) ( ) gmmT 21 ⋅−= ; ( )

1

21

m

gmma

⋅−=

(e) ( ) gmmT 21 ⋅= + ; ( )

1

21

m

gmma

⋅+=

15. (UFPI) A figura mostra dois blocos sobre uma mesa lisa plana e horizontal. As massas dos blocos são m1 = 2 kg e m2 = 8 kg. Ao sistema é aplicada uma força F, horizontal, de intensidade 40 N. A intensidade da força que o bloco m1 exerce sobre o bloco m2 é:

(a) 4 N (b) 8 N (c) 24 N (d) 32 N (e) 40 N

16. (F.C. Chagas – SP) Quatro blocos, M, N, P e Q, deslizam sobre uma

superfície horizontal, empurrados por uma força Fr

, conforme esquema abaixo. A força de atrito entre os blocos e a superfície é desprezível, e a massa de cada bloco vale 3,0 kg. Sabendo-se que a aceleração escalar dos blocos vale 2,0 m/s2, a força do bloco M sobre o bloco N é, em newtons, igual a:

(a) 0 (b) 6 (c) 12 (d) 18 (e) 24

17. (UFES) Desprezando-se os atritos, a aceleração do bloco A será de: (a) 12,0 m/s2 (b) 9,8 m/s2 (c) 4,8 m/s2 (d) 4,0 m/s2 (e) 2,4 m/s2

18. (Fatec – SP) A equação horária da velocidade de uma partícula em movimento retilíneo e de 3 kg de massa é v = 4 + 2� t, com unidades do Sistema Internacional. A força resultante sobre a partícula tem módulo de:

(a) 6 N (b) 2 N (c) 30 N (d) 3 N (e) 1,5 N

19. (FEI – SP) Sabendo-se que a tração no fio que une os dois blocos vale

100N, qual é o valor do módulo da força Fr

? Não há atrito.

20. (CESGRANRIO – RJ) Dois corpos de pesos respectivamente iguais a 20N e 30N são mantidos em equilíbrio, como mostra a figura. P representa um dinamômetro de massa desprezível. Qual a indicação do dinamômetro?

(a) 50 N (b) 30 N (c) 20 N (d) 10 N (e) zero

21. (UNIMEP–SP) Um corpo A de massa 1600 gramas está unido por um fio a um outro corpo B de massa 400 gramas, numa região em que g = 10 m/s2. No instante inicial, o corpo A tinha uma velocidade de 5 m/s e se movia para direita, conforme o esquema. Desprezando-se os atritos, após 5s, o módulo e o sentido da velocidade de A serão:

(a) v = 5m/s; da esquerda para direita. (b) v = 0m/s; da esquerda para direita. (c) v = 0 m/s; da direita para esquerda. (d) v = 5m/s; da direita para esquerda. (e) v = 2m/s; da esquerda para direita.

22. (FATEC–SP) No sistema esquematizado da figura, os blocos A e B têm massas respectivamente iguais a mA e mB, o fio é ideal e não há atritos. Sendo g a aceleração da gravidade e T a tração no fio quando o sistema está em movimento, podemos afirmar que:

(a) T = mA � g (b) T = mB � g (c) T > mB � g (d) T = (mA + mB) � g (e) T < mA � g

D E Fr

m1

m2

m1

m2

Fr

N

Fr

M

Q

P

A

B

N 24F =r

Parede

3 kg

2 kg

Fr

Par 5 kg

10 kg

30 N 20 N

P

A

v0 = 5m/s

B

B

A

FÍSICA – MECÂNICA

AUTORIA – PROF. MARCELO CORREIA E-mail: [email protected] 5

23. (PUC–SP) O esquema representa dois corpos A e B de massas respectivamente igual a 8,0 kg e 2,0 kg, ligados por um fio inextensível e de massa desprezível. No instante t = 0 os corpos estão em repouso na posição indicada no esquema. Nesse instante abandona-se o sistema, que assume movimento devido à tração exercida por B. Despreze as forças de atrito e suponha que a aceleração da gravidade tem intensidade 10 m/s2. O tempo que A leva para ir de M até N é:

(a) 1,0 s

(b) 2 s (c) 2,0 s

(d) 5 s (e) 3,0 s

24. (Fund. Carlos Chagas – SP) A figura mostra um sistema de roldanas sustentando uma lâmpada. Os atritos e as massas das roldanas e das cordas são desprezíveis. A lâmpada L, cujo peso é P newtons, é equilibrada pelo peso X, cujo valor em newtons é:

(a) P/2 (b) P (c) 2� P (d) 4� P (e) 6� P

25. (FEI – SP) No sistema da figura ao lado, o fio e as polia são ideais. Qual a

relação

B

A

a

a entre as

acelerações adquiridas pelos corpos A e B, sabendo que as massa obedecem à relação

4

1

m

m

B

A = ? Despreze o

atrito.

26. (CESCEA – SP) Dois corpos A e B, de massas respectivamente iguais a 2kg e 4kg, estão encostados um no outro e podem se deslocar sem atrito

sobre um plano horizontal. Sobre o corpo A é aplicada a força AFr

de

módulo 12N e sobre o corpo B é aplicada a força BFr

de módulo 6N,

conforme a figura. A aceleração do conjunto vale, aproximadamente: (a) 3,0 m/s2 (b) 1,41 m/s2 (c) 1,0 m/s2 (d) 0,41 m/s2 (e) zero

27. (PUC – SP) Um bloco apoiado sobre uma superfície horizontal sem atrito está inicialmente em repouso. A seguir, aplica-se ao bloco uma força de intensidade igual à metade de seu peso, numa direção que forma um ângulo θ com a horizontal. O valor de θ para que o bloco entre em movimento é necessariamente:

(a) zero. (b) Menor que 30º. (c) Menor que 45º. (d) Diferente de 90º. (e) O bloco não entra em movimento qualquer que será θ.

28. (ITA – SP) Um vagão desloca-se horizontalmente, em linha reta, com aceleração a

r constante. Um pêndulo simples está suspenso do teto do

vagão, sem oscilar e formando ângulo θ com a vertical. Sendo g a aceleração da gravidade e m a massa do pêndulo. O módulo da tensão T no fio do pêndulo é:

(a) cosθgmT ⋅⋅= (b) senθamT ⋅⋅=

(c) 22 gamT +⋅= (d) ( )senθacosθgmT ⋅−⋅⋅=

(e) ( )cosθasenθgmT ⋅+⋅⋅=

29. (UFGO) Um bloco desliza sobre um plano horizontal sem atrito com velocidade constante 0v

r. Em seguida, ele sobre uma rampa de inclinação

θ, também sem atrito, até parar no ponto C da figura. A distância BC percorrida ao longo da rampa é:

(a) tgθg2

v 20

⋅⋅ (b)

cosθg2

v 20

⋅⋅ (c)

senθg

v2 20

⋅⋅

(d) tgθg

v 20

⋅ (e)

senθg2

v 20

⋅⋅

30. (Mack – SP) Os corpos A (mA = 2,0 kg) e B (mB = 4,0 kg) da figura abaixo

sobem a rampa com movimento uniforme, devido à ação da força Fr

, paralela ao plano inclinado. Despreze os atritos e adote g = 10 m/s2. A intensidade da força que A exerce em B é de:

(a) 2,0 N (b) 3,0 N (c) 20 N (d) 30 N (e) 40 N

31. (ITA – SP) O plano inclinado da figura tem massa M e sobre ele se apóia um objeto de massa m. O ângulo de inclinação é αααα e não há atrito nem entre o plano inclinado e o objeto, nem entre o plano inclinado e o apoio horizontal.

Aplica-se uma força Fr

horizontalmente ao plano inclinado e constata-se que o sistema todo se move horizontalmente sem que o objeto deslize em relação ao plano inclinado. Podemos afirmar que, sendo g a aceleração da gravidade local:

(a) F = m�g (b) F = (M + m)� g (c) F tem que ser infinitamente grande (d) F = (M + m)� g�tgα (e) F = Mg�senα

32. (PUC – SP) Dois blocos A e B, de pesos respectivamente iguais a 30N e 70N, apóiam-se sobre uma mesa horizontal, ligados por um fio ideal. O coeficiente de atrito entre os blocos e a mesa é 0,4 e a aceleração da

gravidade é g = 10 m/s2. Aplicando-se ao bloco A uma força horizontal Fr

de intensidade 50 N, a aceleração comunicada ao sistema é:

(a) 5 m/s2 (b) 4 m/s2 (c) 3 m/s2 (d) 2 m/s2 (e) 1 m/s2

A

B

1,0 m solo

5 m

M

N

X L

A

B

B

A

BFr

AFr

45º

0vr

θ

B

C

A

B

30º

Fr

m

M αααα

Fr

Fr

B

A

FÍSICA – MECÂNICA

AUTORIA – PROF. MARCELO CORREIA E-mail: [email protected] 6

33. (PUC – SP) Para o caso da questão anterior, a tração no fio vale: (a) 50 N (b) 35 N (c) 25 N (d) 15 N (e) 10 N

34. (Fund. Carlos Chagas – SP) Um corpo de massa igual a 4,0 kg desloca-se sobre uma superfície plana horizontal, ao longo de uma linha reta, com velocidade escalar constante e igual a 2,0 m/s. O corpo se move sob a ação de uma força constante cuja direção é paralela à trajetória do corpo e cuja intensidade é 3,0 newtons. Podemos afirmar que o módulo da força de atrito entre o corpo e a superfície é:

(a) 3,0 N (b) 5,0 N (c) 6,0 N (d) 8,0 N (e) 11,0 N

35. (FEI – SP) Lança-se um corpo num plano horizontal com velocidade v0=10m/s. O corpo desloca-se sobre o plano e pára após 10s. Dado g=10m/s2, calcule o coeficiente de atrito entre o bloco e a superfície.

(a) 0,1 (b) 0,2 (c) 0,3 (d) 0,4 (e) 0,5

36. (PUC – SP) No sistema da figura, os corpos A e B têm massas mA = 6 kg e mB = 4 kg, respectivamente. O fio que os une e a polia são ideais. A resistência do ar é desprezível, a aceleração local da gravidade é g=10m/s2, e o coeficiente de atrito entre o corpo A e o plano horizontal é µµµµ. Quando o sistema é abandonado em repouso na posição indicada, os blocos adquirem aceleração de 1m/s2. Nessas condições, podemos afirmar que o valor de µµµµ é:

(a) 0,10 (b) 0,25 (c) 0,30 (d) 0,50 (e) 0,75

37. (FATEC – SP) Uma caixa desliza ao longo de um plano inclinado com atrito e inclinação θ em relação a horizontal. Ao ser aumentado o ângulo θ, a força de atrito:

(a) Não se altera. (b) Aumenta de intensidade. (c) Muda de sentido mas não de intensidade. (d) Diminui de intensidade. (e) Nenhuma das anteriores.

38. (FM ABC – SP) Um bloco de metal é colocado sobre uma mesa horizontal que se vai inclinando gradualmente. Quando a mesa forma com a horizontal o ângulo θ da figura, o bloco fica na iminência de deslizar. O coeficiente de atrito, entre o bloco e a mesa, vale:

(a) 0,20 (b) 0,30 (c) 0,40 (d) 0,10 (e) 0,70

39. (FATEC – SP) Com pára-quedas aberto, um soldado salta de um helicóptero em grande altura acima de uma planície. Sobre o sistema formado pelo pára-quedas e pelo homem, podemos afirmar que:

(a) a velocidade cresce uniformemente com aceleração inferior a g. (b) a velocidade de chegada ao solo depende da altura inicial. (c) A velocidade de chegada ao solo depende da duração do processo. (d) À medida que a velocidade se eleva, aumenta a força resultante que as

cordas exercem no homem. (e) Nenhuma das anteriores.

40. (ITA – SP) Numa região em que g = 10m/s2, um corpo de massa m = 8,0 gramas cai na água, atingindo após alguns segundos uma velocidade praticamente constante de 5,0m/s. Sabe-se que, neste caso, a força de resistência exercida pela água é dada pro Fr = k� v, onde v é a velocidade do corpo. Desprezando o empuxo da água, podemos afirmar que a constante k é igual a:

(a) 16 N� s/m (b) 1,6� 102 kg/s (c) 1,6� 103 kgf/s (d) 1,6� 10–3 N� s/m (e) N.d.a.

41. (FEI – SP) No sistema da figura, o corpo A tem peso 200 N, as molas M1 e M2 possuem constantes elásticas k1 = 103 N/m e k2 = 2� 103 N/m. As molas e as polias são ideais. As deformações produzidas nas molas M1 e M2 valem, respectivamente:

(a) 10 cm e 5 cm (b) 20 cm e 0 cm (c) 20 cm e 10 cm (d) 10 cm e 10 cm (e) N.d.a.

42. (FATEC – SP) A figura indica um corpo A de 4 kg preso na extremidade de uma mola, de constante elástica 100 N/m, apoiado numa mesa. Nestas condições a mola experimenta um aumento de comprimento de 10 cm. Considerando-se g = 10m/s2, podemos afirmar que a mesa exercerá sobre o corpo A uma força de intensidade:

(a) 40 N (b) 30 N (c) 20 N (d) 10 N (e) 00 N

43. (FEI – SP) Os corpos A e B representados na figura possuem, respectivamente, massas mA = 2,0 kg e mB = 4,0 kg. A mola é ideal e tem

constante elástica k = 50 N/m. Despreze os atritos. Aplicando a força Fr

constante e horizontal, verifica-se que a mola experimenta deformação de 20cm. Calcule as intensidades: I. Da aceleração do conjunto;

II. Da força Fr

.

44. (VUNESP – SP) Uma pedra de massa m = 0,20 kg gira, presa a um fio, descrevendo uma circunferência horizontal de raio R = 20 cm, enquanto perfaz 2,0 rotações por segundo. Tentando aumentar a velocidade angular, vermos que o fio se rompe. Calcule a tração máxima que o fio suporta (g = 10 m/s2).

(a) 10 N (b) 6,0 N (c) 6,3 N (d) 2,0 N (e) 6,6 N

45. (CESCEM – SP) Uma esfera de massa 0,50 kg oscila no vácuo suspensa por um fio de 1 m de comprimento. Ao passar pela parte mais baixa da

trajetória, ela tem velocidade de 10 m/s. Aceleração da gravidade g=10m/s2. O valor da intensidade da força de tração no fio, na parte mais baixa da trajetória, é um valor expresso em newtons, igual a:

(a) 10,0 (b) 8,0 (c) 7,5 (d) 5,0 (e) zero

46. (OSEC – SP) Um motociclista descreve uma circunferência vertical num “globo da morte” de raio 4 m. Que força é exercida sobre o globo no ponto mais alto da trajetória se a velocidade da moto é ali de 12 m/s? A massa total (motociclista + moto) é de 150 kg.

(a) 1500 N (b) 2400 N (c) 3900 N (d) 5400 N (e) 6900 N

A

B

15 cm

50 cm

θ

A

M2

M1

mesa

A

A

B

Fr

FÍSICA – MECÂNICA

AUTORIA – PROF. MARCELO CORREIA E-mail: [email protected] 7

47. (UFPA) Um pequeno corpo de massa m está preso à extremidade de um acorda de comprimento L e gira com velocidade angular ω em uma circunferência vertical. A tração na corda, quando o corpo está no ponto mais alto da trajetória é, dada por:

(a) ( )gLωm2 +

(b)

− g

4

L2ωm

(c) gL2ωm −

(d) ( )gLωm2 − (e) gL2ωm +

48. (UnB – DF) Um certo trecho de uma montanha-russa é aproximadamente um arco de circunferência de raio R. Os ocupantes de um carrinho, ao passar por este trecho, sentem uma sensação de aumento de peso. Avaliam que, no máximo, o seu peso foi triplicado. Desprezando os efeitos de atritos, os ocupantes concluirão que a velocidade máxima atingida foi de:

(a) 3gR (b) gR3 (c) gR2 (d) 2gR (e) gR

49. (EE MAUA – SP) Numa estrada existe uma curva circular plana de raio 150m. O coeficiente de atrito lateral entre o pneu e a estrada é 0,15. Determine a maior velocidade com que o carro pode percorrer a curva sem derrapar. (g = 10 m/s2).

50. (OSEC – SP) Um toca-discos tem o prato na posição horizontal e realiza 3 revoluções em ππππ segundos. Colocando-se uma pequena moeda sobre o prato, ela deslizará se estiver a mais de 10 cm do centro. Então, o coeficiente de atrito estático entre a moeda e o prato é de:

(a) 0,12 (b) 0,24 (c) 0,36 (d) 0,48 (e) n.d.a.

51. (FEI – SP) Um esfera gira com velocidade 1 m/s, descrevendo uma trajetória circular, horizontal, de raio R = 10 cm. Estando a esfera suspensa por meio de um fio. Qual o ângulo que este fio forma com a vertical? Adote g = 10 m/s2.

52. (PUC – SP) O raio de uma curva ferroviária é de 400 metros e um trem deve percorrê-la com velocidade de 72 km/h. De quanto deve estar elevado o trilho externo para reduzir a um mínimo a força para fora sobre ele? A distância entre os trilhos é de 1,2 metros e g = 10 m/s2.

(a) 0,20m (b) 0,12m (c) 0,15m (d) 0,18m

53. (Mackenzie – SP) Admitamos que você esteja apoiado, em pé, sobre o fundo de um cilindro de 4 m de raio, que gira em torno do seu eixo vertical. Admitindo g = 10 m/s2 e o coeficiente de atrito entre a sua roupa e a superfície do cilindro igual a 0,4 a mínima velocidade tangencial que o cilindro deve ter para que, retirado o fundo do mesmo, você fique “preso” à parede dele, é:

(a) 10 m/s (b) 8 m/s (c) 9 m/s (d) 11 m/s (e) é necessário conhecer a sua massa, pois sem ela nada se pode afirmar.

54. Um homem de massa mH = 80 kg está sobre uma balança de molas, a qual está fixa no piso de um elevador. A massa do elevador juntamente com a balança é mE = 520 kg. O conjunto está inicialmente em repouso. A partir de determinado instante, aplica-se ao teto do elevador, através do cabo que sustentação, uma força vertical para cima de modo que o elevador começa a subir com movimento acelerado,cuja aceleração tem módulo a = 2 m/s2. I. Calcule a intensidade da força aplicada ao teto do elevador; II. Calcule a marcação da balança.

55. O sistema esquematizado na figura está inicialmente em repouso. O fio e a polia são ideais, g = 10 m/s2, a massa de A é 4,0 kg e a de B é 16 kg. Existe atrito entre A e B e entre B e a superfície de apoio, sendo o coeficiente de atrito dinâmico igual a 0,20 em ambos os casos. A partir de determinado instante, aplica-se ao bloco B uma força

horizontal Fr

, como mostra

figura. Calcule o módulo de Fr

nos seguintes casos: I. Os blocos passam a mover-se com velocidade constante; II. Os blocos passam a mover-se com aceleração constante de

módulo a = 3,0 m/s2.

BRINCANDO COM A COVEST (UFPE – UFRPE)

56. A figura abaixo representa uma polia sem massa e sem atrito. Os corpos de massa mA = 4 kg e mB = 1 kg estão presos a uma corda inextensível e de massa desprezível. Qual o módulo da aceleração do corpo de massa mA, em m/s2?

57. No plano inclinado da figura seguinte, o bloco de massa M desce com aceleração a = 2 m/s2, puxando o bloco de massa m. Sabendo-se que não há atrito de qualquer espécie, qual o valor da razão

mM ? g = 10 m/s2.

58. Uma pessoa puxa um bloco de massa 0,2 kg com auxílio de uma mola de constante elástica igual 20 N/m, conforme a figura. Se o coeficiente de atrito estático entre o bloco e a superfície horizontal é 0,5, qual o alongamento máximo da mola, em cm, que ainda mantém o bloco em repouso?

59. Uma gota de chuva, de massa igual a 0,05 g, cai verticalmente com velocidade constante. Qual a força de resistência do ar, atuando sobre a gota, em unidades de 10–5N?

60. No sistema mostrado na figura, o bloco tem massa igual a 5,0 kg. A constante elástica da mola, 2,0 N/m. Considere que o fio, a mola e a roldana são ideais. Na situação de equilíbrio, qual a deformação da mola, em cm?

61. A figura mostra dois blocos em repouso. O coeficiente de atrito estático entre o bloco B, de massa 30kg, e a superfície de apoio é 0,6. considere que a polia e o fio são ideais. Qual o maior valor, em kg, da massa do bloco A para que o sistema permaneça em repouso?

62. Um bloco de 6,0kg sobe o plano inclinado na figura, sob a ação de uma força externa paralela ao plano. O coeficiente de atrito entre o plano e o

bloco é 2

3µ = . Qual será o valor da força externa F, em newtons, para

que o bloco esteja em movimento uniforme?

63. Um objeto de 2,0kg descreve uma trajetória que obedece à equação horária S = 5,0 + 3,0� t + 7� t2, onde S é medido em metros e t em segundos. O módulo da força resultante que está atuando sobre o objeto é, em N.

(a) 10 (b) 17 (c) 19 (d) 28 (e) 35

Fr

A

B

mA

mB

M

m

30º

A

B

30º

Fr

FÍSICA – MECÂNICA

AUTORIA – PROF. MARCELO CORREIA E-mail: [email protected] 8

64. Um físico, atendendo à sua esposa, tenta mudar a localização da sua geladeira empurrando-a horizontalmente sobre o chão, mas não consegue movê-la. Pensando sobre o assunto, ele imagina como sua vida seria mais fácil num planeta de gravidade menor que a da Terra. Considerando que a força que o físico faz sobre a geladeira vale 1200N, a massa da geladeira é 300kg, e o coeficiente de atrito estático entre a geladeira e o chão é ½, indique entre os planetas abaixo aquele com maior aceleração da gravidade, g, no qual ele ainda conseguiria mover a geladeira.

(a) Plutão, g = 0,3 m/s2 (b) Marte, g = 3,7 m/s2 (c) Urano, g = 7,8 m/s2 (d) Vênus, g = 8,6 m/s2 (e) Saturno, g = 9,0 m/s2

65. Um corpo de massa igual a 10,0 kg é suspenso em uma balança de mola presa ao teto de um elevador. Calcule a diferença, em newtons, entre as leituras na balança quando o elevador está subindo ou descendo, sabendo que nos dois casos a aceleração tem módulo a = 0,5 m/s2.

66. Uma partícula inicialmente em repouso é submetida à ação da força mostrada no gráfico abaixo. Indique qual dos gráficos melhor representa a variação da posição da partícula em função do tempo.

(a)

(b)

(c)

(d)

(e)

67. Um homem sobe numa balança no interior de um elevador. Com o elevador parado a indicação da balança é de 60kg. Se o elevador estiver subindo com aceleração de 2 m/s2, qual será a indicação da balança. (g = 10 m/s2)

(a) 48 kg (b) 60 kg (c) 72 kg (d) 84 kg (e) 96 kg

68. Dois blocos A e B de massas respectivamente iguais a 5 kg e 10 kg está inicialmente em repouso, encostados um no outro, sobre uma mesa horizontal sem atrito. Aplicando-se uma força horizontal F = 90 N, como

mostra a figura. Os valores em N, das forças resultantes que atuam sobre os blocos A e B são respectivamente:

(a) 40 e 50 (b) 45 e 45 (c) 90 e 90 (d) 20 e 70 (e) 30 e 60

69. Um pequeno bloco de 0,50 kg desliza sobre um plano horizontal sem atrito, sendo puxado por uma força constante F = 10,0 N aplicada a um fio inextensível que passa por uma roldana, conforme a figura. Qual a aceleração do bloco, em m/s2, na direção paralela ao plano, no instante em que ele perde o contanto com o plano? Despreze as massas do fio e da roldana, bem como o atrito no eixo da roldana.

(a) 12,4 (b) 14,5 (c) 15,2 (d) 17,3 (e) 18,1

70. Um bloco de massa 1,5 kg é solto, a partir do repouso, do topo de um plano inclinado de 5,0 m de altura, conforme a figura. O tempo gasto pelo bloco para descer até a base do plano é igual a 2,0 s. Qual o comprimento do plano inclinado, em metros? Despreze o atrito entre o bloco e o plano.

BRINCANDO COM A UPE

71. Um bloco de massa m = 5 kg esta subindo a rampa inclinada de 30º com a horizontal, mantendo a velocidade constante. O atrito vale 40% do peso do bloco. A força F tem módulo igual a:

(a) 25 N (b) 30 N (c) 35 N (d) 40 N (e) 45 N

72. O caminhão altera a velocidade altera a velocidade de 54 para 90 km/h num tempo de 10 segundos. O caixão de 1000 kg não desliza sobre a carroceria. Qual a força de atrito, em kgf, na superfície de contato entre o caminhão e o caixote?

(a) 100 (b) 80 (c) 40 (d) 60 (e) 20

73. No dimensionamento de um novo automóvel, pesando 1600kgf, estabeleceu-se que o rebaixamento máximo das molas com a carga máxima de 5 pessoas de 80kg é de 5cm. Qual deve ser a constante elástica (em N/m) comum às quatro molas?

(a) 50000 (b) 40000 (c) 30000 (d) 20000 (e) 10000

74. Uma pessoa comprou uma balança de chão e, ao chegar em casa, ansiosa para controlar o peso, resolve testá-la ainda no elevador. Ela conclui que a balança estava com defeito ao notar um aumento de seu peso. Considerando essas informações, identifique a opção correta.

(a) O aumento da indicação da balança pode ocorrer se o elevador está subindo com velocidade constante.

(b) O aumento da indicação da balança pode ocorrer se o elevador está descendo com velocidade constante.

(c) O aumento da indicação da balança pode ocorrer se o elevador está subindo com aceleração constante.

(d) O aumento da indicação da balança pode ocorrer se o elevador está descendo com aceleração constante.

(e) A balança está necessariamente com defeito e deve ser trocada em respeito aos direitos do consumidor.

F0

– F0

x

t

t0

2t0

F0

– F0

F

t

t0

2t0

x

t

t0

2t0

x

t

t0

2t0

x

t

t0

2t0

x

t

t0

2t0

A

B

Fr

F

H = 5,0 m

30º

Fr

FÍSICA – MECÂNICA

AUTORIA – PROF. MARCELO CORREIA E-mail: [email protected] 9

75. Um corpo de massa m está em repouso sobre um plano inclinado que faz um ângulo de 30º com a horizontal, conforme mostra a figura. Em relação a essa situação, analise as seguintes proposições.

I II

0 0

A força normal que o plano inclinado exerce sobre o corpo de massa m está na direção vertical, no sentido de baixo para cima.

1 1 A força de atrito é maior do que m� g. 2 2 A força peso é perpendicular ao plano inclinado. 3 3 A força de atrito é igual a m� g� sen30º. 4 4 A força normal é igual a m� g� cos(a/b).

76. Uma menina está no carro de uma montanha-russa, que faz uma volta circular completa na vertical. No topo da trajetória, a força normal exercida pela cadeira sobre a menina é igual a duas vezes o peso da menina, 2mg. No ponto mais baixo da trajetória, a força normal exercida pela cadeira sobre a menina é:

(a) Menor do que o peso da menina. (b) Igual ao peso da menina. (c) Igual à força normal no topo da trajetória. (d) Igual a quatro vezes o peso da menina. (e) Igual a oito vezes o peso da menina.

77. Em relação ao conceito e ao tipo de força. I II 0 0 As forças de ação e reação sempre atuam em corpos distintos. 1 1 A força elástica é proporcional à deformação da mola. 2 2 A força normal é uma força de reação ao peso. 3 3 Força é uma grandeza vetorial. 4 4 Uma força sempre causa mudança no valor da velocidade.

ANOTAÇÕES / OBSERVAÇÕES

DINÂMICA – PARTE 2 Nesta segunda parte do estudo da dinâmica vamos estudar grandezas físicas muito importantes: Trabalho e potência, energia. Vamos discutir um princípio de conservação muito importante: O princípio de conservação da energia mecânica.

TRABALHO ττττ Em Física a palavra trabalho não tem exatamente o mesmo significado que usamos cotidianamente. Em Física o trabalho é realizado quando uma força “ajuda” ou “atrapalha” um deslocamento. Vamos considerar um corpo de massa m que efetue um deslocamento para direita com módulo d, onde no mesmo atua uma força constante com módulo F, como mostra a figura a seguir:

Nestas condições, se a força Fr

que atua no corpo durante um

deslocamento dr

for constante podemos calcular o trabalho efetuando o produto escalar entre o vetor força e o vetor deslocamento, assim temos:

dFrr

•=τ Mas, como já sabemos da álgebra vetorial um produto escalar entre dois vetores pode ser calculado por:

θτ cosdF ⋅⋅= Se analisarmos a expressão anterior podemos perceber facilmente que:

• 0º ≤≤≤≤ θθθθ < 90º o trabalho realizado pela força Fr

é positivo, neste caso chamamos este trabalho de trabalho motor e isto significa que a força “ajuda” o deslocamento. Neste caso a força transfere energia para o corpo;

• θθθθ = 90º o trabalho realizado pela força Fr

é nulo, neste caso a força nem “ajuda” nem “atrapalha” o deslocamento do corpo;

• 90º < θθθθ ≤≤≤≤ 180º o trabalho realizado pela força Fr

é negativo, neste caso chamamos este trabalho de trabalho resistente e isto significa que a força “atrapalha” o deslocamento. Neste caso a força retira energia do corpo. Você percebe que o trabalho é uma grandeza que indica a

transformação de energia. Quando uma força realiza trabalho ela pode transformar energia de uma forma em outra e o sistema sobre o qual o trabalho está sendo realizado pode está recebendo energia ou pode está sendo retirada energia do sistema.

A unidade de trabalho no SI é o N� m (Newton vezes metro) que recebe o nome especial de J (joule), assim: 1 N� m = 1 J.

Podemos destacar três situações tem grande freqüência de aparição nas questões a serem resolvidas:

• Quando a força tem mesma direção e mesmo sentido do deslocamento: Neste caso podemos calcular o trabalho pela

expressão: dF ⋅=τ ; • Quando a força tem mesma direção e sentido oposto ao

deslocamento: Neste caso podemos calcular o trabalho pela

expressão: dF ⋅−=τ • Quando a força é perpendicular ao deslocamento: Neste caso não

a realização de trabalho, isto é: 0=τ . É importante lembrar que se num corpo atuar diversas forças cada

força realiza o seu trabalho independente das demais forças e o trabalho realizado pela força resultante é a soma algébrica dos trabalhos realizados por todas as forças individualmente.

a

b

30º

d

θθθθ Fr

dr

FÍSICA – MECÂNICA

AUTORIA – PROF. MARCELO CORREIA E-mail: [email protected] 10

Trabalho Realizado pela Força Peso Aplicando a definição de trabalho realizado por uma força mostra anteriormente, podemos calcular o trabalho realizado pela força peso. Assim podemos observar duas situações em que a força peso realiza trabalho:

• Na descida de um corpo de uma determinada altura “h”: Neste caso constatamos que o trabalho realizado pode ser calculado pela

expressão: hP ⋅=τ ou hgm ⋅⋅=τ ;

• Na subida de um corpo a uma determinada altura “h”: Neste caso constatamos que o trabalho realizado pode ser calculado pela

expressão: hP ⋅−=τ ou hgm ⋅⋅−=τ .

Gráfico Fx S

No gráfico da força F em função da posição S a área limitada pelo gráfico e o eixo das posições é numericamente igual ao trabalho realizado pela força no deslocamento d considerado. Quando uma força variar com a posição a expressão:

θτ cosdF ⋅⋅= , que usamos para calcular o trabalho não poderá ser usada pois esta expressão só é válida para o caso em que a força é constante. Quando a força variar com a posição podemos calcular o trabalho recorrendo ao gráfico da força em função da posição como descrito acima.

Trabalho Realizado pela Força Elástica A força elástica é uma força que varia com a posição. Portanto par calcularmos o trabalho realizado pela força elástica temos que recorrer ao gráfico da força elástica em função da posição, que neste caso é a deformação “x” da mola. Fazendo isto obtemos a seguinte expressão:

2

xk 2⋅±=τ

Onde “k” é a constante elástica (ou constante de força) da mola, “x” é a deformação da mola. O sinal mais o menos é determinado observando-se se a força elástica se opõe (caso em que usamos menos) ou favorece o deslocamento (caso em que usamos mais).

POTÊNCIA ( Pot ) Imagine duas pessoas que realizam o mesmo trabalho. Porém, uma delas realiza o trabalho num intervalo de tempo menor que a outra. Então, a de se pensar que a pessoa mais rápida é mais eficiente do que a que realizou o mesmo trabalho num intervalo de tempo maior. Para diferenciar a com o trabalho está sendo realizado em relação ao tempo usamos a grandeza física potência. Portanto, a potência média é a relação entre o trabalho realizado e o intervalo de tempo para se realizar este trabalho. Assim temos:

tPot ∆

Efetuando algumas transformações matemáticas e lembrando a definição de velocidade podemos escrever a potência como:

vFPot

rr

•= ou θcosvFPot ⋅⋅=

Onde θθθθ é o ângulo formado entre o vetor força e o vetor velocidade do corpo. Para o caso especial em que a força e a velocidade têm mesma direção e

mesmo sentido podemos escrever: vFPot ⋅= .

A unidade de potência no sistema internacional (SI) é o J/s (joule por segundo) que recebe o nome especial de W (watt), assim temos: 1 J/s = 1 W. Outras unidades de potência usadas são o: HP (horse-power) e o CV (cavalo-vapor). As relações são: 1 CV = 735 W; 1 HP = 745,7 W ≈≈≈≈ 746 W.

Gráfico da Potência em Função do Tempo No gráfico da potência em função do tempo a área limitada pelo gráfico e o eixo dos tempos é numericamente igual ao trabalho realizado no intervalo de tempo considerado.

RENDIMENTO ηηηη Imagine uma máquina qualquer que deve realizar determinado trabalho. Digamos que seja fornecida certa energia para que esta máquina realize tal trabalho, no entanto, parte desta energia é desperdiçada por forças dissipativas, tal como a força de atrito. O rendimento é uma grandeza física que mede a eficiência de uma máquina, isto é, o rendimento mede quanta da energia fornecida a uma máquina é usada por ela para realizar um certo trabalho útil e, conseqüentemente mede quanto ela dissipa usando para realizar trabalho em forma não útil. Assim, quanto maior for o rendimento de uma máquina mais eficiente ele é. Podemos então destacar três tipos de trabalho ou potências:

• Trabalho útil • Potência útil • Trabalho dissipado • Potência dissipada • Trabalho total • Potência total

De forma que podemos escrever:

duT τττ += e duT PPP +=

Assim o rendimento pode ser dado por:

T

uττη = ou

T

u

P

P=η

Note que o rendimento é uma grandeza adimensional (que não tem unidade) já que é uma relação entre mesmas grandezas. O rendimento é,

sempre, um valor entre zero e um, isto é: 10 ≤≤η . Uma máquina que tenha

rendimento igual a 1 (um) é uma máquina ideal. Podemos expressão o rendimento em percentual. Para obter o rendimento percentual basta multiplicar o rendimento por 100. Assim temos:

ηη ⋅= 100%

BRINQUEDINHO DE VESTIBULANDO!!!!!

78. Um ponto material de massa 6kg tem velocidade de 8m/s quando sobre ele passa a agir uma força de intensidade 30N na mesma direção e sentido do movimento, durante 4s. Determine: I. O deslocamento durante esses 4s; II. O trabalho realizado nesse deslocamento.

79. Um móvel de massa 40kg tem velocidade constante de 90km/h. Num determinado instante entra numa região rugosa onde o coeficiente de atrito é igual a 0,2. Determine: I. O espaço percorrido pelo móvel na região rugosa até parar; II. O trabalho realizado pela força de atrito.

80. Um corpo de massa 10kg é arrastado ao longo de um plano horizontal rugoso cujo coeficiente de atrito vale 0,2 por uma força horizontal de intensidade F = 60N durante 20s. Sabendo que o corpo estava inicialmente em repouso e g = 10m/s2, calcule, nesses 20s. I. O trabalho realizado pela força F e o trabalho da força de atrito;

F

S

d

ττττ = A 0

FÍSICA – MECÂNICA

AUTORIA – PROF. MARCELO CORREIA E-mail: [email protected] 11

II. O trabalho realizado pela força peso e pela normal; III. O trabalho realizado pela força resultante.

81. Determine o trabalho realizado por uma força de 200N num deslocamento de 8m nos seguintes casos: I. Força e deslocamento formam um ângulo de 30º; II. Força e deslocamento formam um ângulo de 90º; III. Força e deslocamento formam um ângulo de 120º.

82. Um garoto abandona uma pedra de 0,4kg do alto de uma torre de 25m de altura. Dado g = 10m/s2, calcule o trabalho realizado pela força peso até a pedra atingir o solo.

83. Uma partícula é lançada obliquamente no vácuo descrendo uma trajetória parabólica. Sabendo que a altura máxima atingida pela partícula foi de 5m e que no local a aceleração da gravidade vale g = 10m/s2. Nestas condições encontre o trabalho, em joule, realizado pela força peso desde o instante de lançamento até o instante em que partícula passa pelo mesmo nível de lançamento.

(a) Zero (b) 5,0 (c) 50 (d) 100 (e) 200

84. Um bloco sofre um deslocamento j10i15d))r

⋅−⋅= em unidade do SI sobre a

ação de uma força j100i220F))r

⋅−⋅= em unidade do SI. Calcule o trabalho

realizado por esta força no deslocamento considerado.

85. Uma usina hidrelétrica foi construída para aproveitar uma queda-d’água de 20m de altura. Se a vazão da água é de 1,5� 102m3/s, qual a potência disponível, supondo que não haja perdas. Dados: aceleração da gravidade µµµµ = 1,0� 103kg/m3 e aceleração da gravidade g = 10m/s2.

86. (Unicamp–SP) Uma hidrelétrica gera 5,0� 109W de potência elétrica utilizando-se uma queda-d’água de 100m. Suponha que o gerador aproveita 100% da energia da queda-d’água e que a represa coleta 20% de toda a chuva que cai em uma região de 400000km2. Considere que 1 ano tem 32�106 segundos, g = 10m/s2 e µµµµágua = 1,0� 103kg/m3. I. Qual a vazão de água (m3/s) necessária para fornecer a potência

indicada? II. Quantos mm de chuva devem cair por ano nessa região para

manter a hidrelétrica operando na potência indicada?

87. O nosso organismo converte energia química interna, em trabalho e em calor, com uma potência de 100 W, que é denominada a nossa taxa ou potência metabólica. I. Que quantidade de energia dissipamos em 24h? II. A energia que dissipamos é oriunda de alimentos, sendo 1kcal =

4,18kJ. Quantas quilocalorias de alimentos devemos ingerir em 1 semana em devido a taxa metabólica?

BRINCANDO COM A COVEST (UFPE – UFRPE)

88. Um elevador é puxado para cima por cabos de aço com velocidade constante de 0,5m/s. A potência mecânica transmitida pelos cabos é de 23kW. Qual a força exercida pelos cabos?

(a) 5,7� 104N (b) 4,6� 104N (c) 3,2� 104N (d) 1,5� 104N (e) 1,2� 104N

89. Uma usina hidrelétrica de 90MW produz energia elétrica por meio de uma turbina acionada pela água que cai de uma cachoeira cuja altura é 100m. Supondo que não há perdas, calcule o volume de água, em m3, que passa pela turbina em cada segundo.

90. Um bloco de massa m = 1,0g é arremessado horizontalmente ao longo de uma mesa, escorrega sobre a mesma e cai livremente, como indica a figura. A mesa tem comprimento d = 2,0 m e altura h = 1,0 m. Qual o trabalho realizado pelo peso do bloco, desde o instante em que foi arremessada até o instante em que toca o chão?

(a) 1,0� 10–2J (b) 1,5� 10–2J (c) 2,5� 10–2J (d) 4,0� 10–2J (e) 5,0� 10–2J

ENERGIA Uma das grandezas mais importantes da Física é a energia. A energia é uma grandeza escalar que pode aparecer de formas variadas na natureza tais como: térmica, mecânica, luminosa, nuclear, elétrica, etc. podendo alternar sua forma em transformações mútuas. O nosso propósito, agora, é estudar a energia mecânica – aquela associada aos fenômenos reativos a movimento. A energia mecânica pode aparecer em duas formas natureza:

• Energia Cinética ���� Associada ao movimento propriamente dito; • Energia potencial ���� Associada a tendência do corpo vir a adquirir

movimento (energia cinética).

Energia Cinética EC A energia cinética é o tipo de energia mecânica assoada ao movimento, isto é, um corpo tem energia cinética em relação a um determinado referencial se estiver em movimento em relação a este referencial. A energia cinética é dada por:

2C vmE

2

1 ⋅⋅=

Pela própria definição da energia cinética constatamos um teorema muito importante chamado de: teorema do trabalho–energia cinética ou simplesmente teorema da energia cinética. O teorema da energia cinética pode ser enunciado da seguinte forma: O trabalho realizado pela força resultante que atua sobre uma partícula é

igual à variação da energia cinética da partícula. Segundo o teorema da energia cinética podemos escrever:

CE∆=τ ou CiCf EE −=τ

Energia Potencial EP

A energia potencial é um tipo de energia armazenada no corpo, que por esse motivo pode vir a ser transformada em energia cinética e assim sendo em movimento. As energias potenciais mecânicas aparecem quando uma força conservativa (força peso ou força elástica) realiza trabalho. Assim podemos considerar:

• Energia potencial gravitacional – Proveniente da força gravitacional (força peso). Esta é dada por:

hgmEPg ⋅⋅=

• Energia potencial elástica – Proveniente da força elástica (lei de Hooke).

2Pel xkE

2

1 ⋅⋅=

Análogo ao teorema da energia cinética podemos considerar um teorema para a energia potencial no caso de só atuar forças conservativas na partícula, este pode ser enunciado da seguinte forma: O trabalho realizado pelas forças conservativas sobre uma partícula é igual

ao negativo da variação da energia potencial desta partícula. Assim temos:

PE∆−=τ

Energia Mecânica EM

Diante do que foi discutido, podemos falar sobre a energia mecânica. Como vimos temos duas formas possíveis de energia mecânica: energia cinética e energia potencial gravitacional ou elástica, pois bem. A energia mecânica é a soma da energia cinética com a energia potencial. Assim temos:

PCM EEE +=

d

d 2,0m

FÍSICA – MECÂNICA

AUTORIA – PROF. MARCELO CORREIA E-mail: [email protected] 12

Princípio da Conservação da Energia Mecânica Num sistema em que só atua forças conservativas a energia mecânica é conservada, isto é, a energia mecânica é constante. Neste caso podemos escrever que:

0EM =∆ ou 0EE MiMf =− ou MfMi EE =

Teorema do Trabalho–Energia

O teorema do trabalho–energia relaciona o trabalho realizado por forças não–conservativas com a variação de energia cinética e potencial do sistema.

Se num sistema só atua forças conservativas, quando há desaparição de energia cinética esta aparece em forma de energia potencial gravitacional ou elástica e quando há desaparição de energia potencial gravitacional ou elástica esta aparece na forma de energia cinética, assim, para um sistema em que só atua forças conservativas a soma da variação da energia cinética com a variação da energia potencial deverá dá zero.

No entanto, se num sistema em que atua forças não–conservativas (forças dissipativas) a soma da variação da energia cinética com a energia potencial deverá da um valor diferente de zero. O Teorema do trabalho–energia diz que: O trabalho realizado por forças não–conservativas sobre uma partícula é igual a soma da variação da energia cinética com a energia potencial da partícula, isto é, é igual a variação da energia mecânica da partícula.

Observe que a soma da energia cinética com a energia potencial da partícula é na verdade a variação da energia mecânica da partícula. Assim, podemos escrever:

MNC E∆=τ ou PCNC EE ∆+∆=τ

Onde o índice NC na indica não–conservativa.

BRINQUEDINHO DE VESTIBULANDO!!!!!

91. (ITA–SP) Uma partícula é deslocada de um ponto A até outro ponto B, sob a ação de várias forças. O trabalho realizado pela força resultante, nesse deslocamento, é igual à variação da energia cinética da partícula:

(a) Somente se a força for constante. (b) Somente se a força for conservativa. (c) Seja a força conservativa ou não. (d) Somente se a trajetória for retilínea. (e) Em nenhum caso.

92. (FEI–SP) Um corpo de massa m = 30 kg, inicialmente em repouso, é posto em movimento sob a ação de uma força constante e adquire, ao fim de dois minutos, uma velocidade de 72 km/h na direção do da força aplicada. Determine: I. A intensidade da força aplicada ao corpo; II. O trabalho realizado pela referida força ao longo da distância

percorrida pelo corpo.

93. (FEI–SP) Um corpo de massa 1,0kg está parado num plano inclinado. Abandonado, desce e tem a velocidade de módulo 2,0m/s, quando a altura de queda (vertical) é 0,80m. Calcular o trabalho das forças não–conservativas. Dado: g = 10m/s2.

94. (Mackenzie–SP) Uma bomba (B) recalca água à taxa de 0,02m3/s, de um depósito (A) para uma caixa (C) no topo de uma casa. A altura de recalque é 9,2m e a velocidade da água na extremidade do topo de descarga (D) É 4m/s. Considerar g = 10m/s2 e a massa específica da água 1000kg/m3. Desprezar as dissipações de energia. A potência da bomba é:

(a) 2500W (b) 2000W (c) 1500W (d) 1000W (e) 500W

95. (FUVEST–SP) Um corpo está preso nas extremidades de duas molas idênticas, não deformadas, de constante elástica 100 N/m, conforme ilustra a figura. Quando o corpo é afastado de 1,0cm do ponto central:

I. Qual a intensidade da resultante das forças que as molas exercem sobre ele?

II. Qual a energia potencial armazenada nas molas?

96. (ITA–SP) A variação da energia cinética de uma partícula em movimento, num dado referencial inercial, entre dois pontos distintos P e Q é sempre igual: I. À variação da energia potencial entre esses dois pontos. II. Ao trabalho da resultante das forças aplicadas à partícula para

deslocá-la entre esses dois pontos. III. À variação da energia potencial entre esses dois pontos, a menos

de sinal, quando a força resultante aplicada à partícula for conservativa.

(a) Somente I é correta. (b) I e II são corretas. (c) Somente III é correta. (d) II e III são corretas. (e) Somente II é correta.

97. (ITA–SP) Uma partícula P move-se em linha reta em torno do ponto x0. A figura ilustra a energia potencial da part´cula em função da coordenada x do ponto. Supondo que a energia total da partícula seja constante e igual a E, podemos afirmar que:

(a) Nos pontos x1 e x2 a energia cinética da partícula é máxima.

(b) A energia cinética da partícula entre x1 e x2 é constante.

(c) No ponto x0 a energia cinética da partícula é nula.

(d) Nos pontos x1 e x2 a energia cinética da partícula é nula.

(e) Nenhuma das anteriores.

(F.M. Santa Casa–SP) Este enunciado refere-se às três próximas questões. O gráfico representa a energia potencial de um sistema conservativo isolado, em função da distância x. Para x = 0 o sistema possui só energia potencial.

98. Para x = 2cm. (a) O sistema tem 1,0� 102 joules de energia total. (b) O sistema só tem energia cinética. (c) O sistema tem energia cinética igual à energia potencial. (d) O sistema perdeu energia. (e) Nada do que se afirmou é correto.

B

A

9,2m

C D

0

1� 102

2

1 3 4

5

6

7

8

EP (J)

|x| (cm)

2M 102

–1� 102

0

x0

x1

Energia

x

E

x2

EP

FÍSICA – MECÂNICA

AUTORIA – PROF. MARCELO CORREIA E-mail: [email protected] 13

99. Marque a correta: (a) Entre 5cm e 7cm o sistema executa um movimento circular. (b) Para |x| maior que 8cm, a energia cinética do sistema é igual a 0,5� 102 J. (c) Para x = 6cm o sistema tem certamente energia cinética menor que 2� 102 J. (d) O sistema perdeu energia. (e) Nada do que se afirmou é correto nas afirmativas anteriores.

100. Para x = 2cm e x = 6cm: (a) As energias potenciais são iguais em valor absoluto. (b) As energias cinéticas são iguais. (c) A soma das energias cinética e potencial variou. (d) As energias cinética são iguais em módulo. (e) Nada do que se afirmou é correto.

101. (Fuvest–SP) Uma bola de 0,2kg é chutada para o ar. Sua energia mecânica, em relação ao solo, vale 50J. Quando está a 5 m do solo, o valor de sua velocidade é: Dado g = 10m/s2.

(a) 5m/s (b) 10m/s (c) 50 m/s (d) 20m/s (e) 100m/s

102. (PUC–SP) Um menino desce um tobogã de altura h = 10m, a partir do repouso. Supondo g = 10m/s2 e que sejam dissipados 50% da energia adquirida na queda, a velocidade do menino ao atingir a base é de:

(a) 210 m/s (b) 10m/s (c) 25 m/s (d) 5m/s (e) 1m/s

103. (Fatec–SP) Na figura, o corpo de 0,2kg é lançado do repouso pela mola M de constante elástica 6� 103 N/m e descreve a trajetória D, E, F, G, H e I sem perder o contanto com a trajetória. Adote g = 10m/s2 e despreze os atritos. A mínima compressão da mola para que isso ocorra é de:

(a) 0,5 cm (b) 1,0 cm

(c) 5 cm (d) 1,0 m (e) 5,0 m

104. Um anel de massa 0,80kg está ligado a uma mola e desliza sem atrito ao longo de um guia circular, situada num plano vertical. A constante elástica da mola é de 40 N/m. Abandona-se o anel em repouso na posição A, determine sua velocidade ao passar pelo ponto B. Sabe-se que o comprimento da mola, quando não deformada, é de 0,40m. Considere a aceleração da gravidade 10m/s2.

(a) 2m/s (b) 20 m/s (c) 2,8 m/s (d) 0,25 m/s (e) 2 km/h

105. Um pêndulo de comprimento L=10dm tem um peso de massa m. O pêndulo é solto fazendo um ângulo θ=60º com a vertical. O fio do pêndulo bate num pino, colocado à uma distância Z=5dm, vertical, do ponto onde está vinculado o pêndulo no teto o que provoca uma diminuição do comprimento do pêndulo. Achar o ângulo máximo β entre o fio e a vertical, quando o peso do pêndulo estiver à direita do pino, conforme a figura.

106. Com que velocidade a esfera deve passar pelo ponto A para chegar ao

ponto B com velocidade de 52 m/s? Sabe-se que no percurso AB houve perda de energia de 20% e g = 10m/s2.

BRINCANDO COM A COVEST (UFPE – UFRPE)

107. Num circo, um motociclista deve percorrer o perímetro interno de um trilho circular vertical, de raio R = 4 m, partindo do repouso, de um ponto com altura H. Se a velocidade do motociclista, ao passar pelo ponto A é de 10 m/s, qual o valor de H, em metros? Despreze o atrito e considere g=10m/s2.

108. Um carrinho escorrega sem atrito em uma montanha russa, partindo do repouso no ponto A, a uma altura H, e sobe o trecho seguinte em forma de um semicírculo de raio R. Qual a razão H/R, para que o carrinho permaneça em contato com o trilho no ponto B?

(a) 5/4 (b) 4/3 (c) 7/5 (d) 3/2 (e) 8/5

109. Um garoto desliza sobre um escorregador, sem atrito, de 5,0 m de altura. O garoto é lançado em uma piscina e entra em contato com a água a uma distância horizontal de 2,0 m, em relação à borda. Calcule a distância vertical h, entre a superfície da água e a borda da piscina. Dê sua resposta em cm.

110. Um bloco cai, a partir do repouso, de uma altura h = 0,9 m acima da extremidade livre de uma mola de constante elástica k= 4,2x103 N/m, como mostra a figura. Se a deformação máxima da mola é x = 0,1 m, qual o peso do bloco, em newtons. Despreze a resistência do ar e a massa da mola.

M

10cm

10cm

D

H

E

F

G

0,40m

0,30m

B

A

L Z

β

θ

3 m B A

R

H

A

H

A B

R

5,0

m

h 2,0 m

h

x K

FÍSICA – MECÂNICA

AUTORIA – PROF. MARCELO CORREIA E-mail: [email protected] 14

111. Deixa-se cair uma bola, a partir do repouso, de uma altura H acima do piso de uma quadra. Após a bola colidir três vezes com o piso, ela se eleva até uma altura H' = H/8. Considerando que a razão entre as energias cinéticas antes e depois de cada colisão, é a mesma, determine o valor desta razão? Despreze a resistência do ar.

112. Um bloco de massa m = 0,1 kg comprime uma mola ideal, de constante elástica k = 100 N/m, de 0,2 m (ver figura). Quando a mola é liberada, o bloco é lançado ao longo de uma pista lisa. Calcule a velocidade do bloco, em m/s, quando ele atinge a altura h = 1,2 m.

113. A figura mostra um bloco de 0,10kg inicialmente forçado contra uma mola de constante elástica k = 480N/m, comprimindo-a de 10cm. Ao soltar, o bloco desliza sobre uma superfície horizontal lisa, exceto no trecho AB, de 50cm, onde o coeficiente de atrito cinético é igual a 0,25. Em seguida o bloco sobe uma rampa sem atrito, retornando posteriormente à superfície horizontal podendo atingir a mola. Quantas vezes o bloco passará pelo ponto A antes de parar completamente?

(a) 20 (b) 9 (c) 18 (d) 24 (e) 25

114. Um brinquedo consiste de duas peças de plástico ligadas através de uma mola. Quando pressionado sobre o solo e abandonado, ele sobre verticalmente na direção da normal. O centro de massa do brinquedo atinge uma altura máxima de 50,0 cm, quando a compressão inicial da mola é de 2,0cm. Se a massa total do brinquedo vale 200g, quanto vale a constante elástica da mola?

(a) 1,0� 103N/m (b) 2,0� 103N/m (c) 3,0� 103N/m (d) 4,0� 103N/m (e) 5,0� 103N/m

BRINCANDO COM A UPE

115. Um carro de 700 kg tem velocidade de 20 m/s quando está em x = 0. Sobre o carro atua uma única força F(x) que varia com a posição, como mostra o gráfico. Em relação a esta situação, analise as propostas abaixo:

I II 0 0 A energia cinética do carro em x = 0, é 140 kJ.

1 1 O trabalho realizado pela força, quando o carro se desloca de x = 0 até x = 50 m, é 175kJ.

2 2 A velocidade do carro em x = 50 m é 30 m/s. 3 3 A energia mecânica é sempre menor do que 140kJ. 4 4 A variação da energia potencial é diferente de zero.

116. Saltando de um helicóptero, estacionário, um pára-quedista ganha 30J de energia cinética após um determinado tempo de queda. Considerando a perda de energia no movimento através do ar, a variação da energia potencial gravitacional, neste mesmo intervalo de tempo, é:

(a) Igual a variação da energia cinética. (b) Menor que 30 J. (c) Maior que 30 J. (d) Igual ao trabalho das forças não conservativas. (e) Sempre igual ao dobro do trabalho das forças não conservativas.

117. Uma menina de massa M é empurrada no topo de um escorregador, partindo com uma velocidade inicial vo de ma altura H, conforme figura ao lado. Desprezando a ação das forças não conservativas, é correto afirmar que a velocidade da menina, ao atingir a altura h, é:

(a) h)2g(H2

0V −+

(b) 2gH

(c) 2gH2

0V +

(d) 2gH0V +

(e) h)2g(H0V −+

118. Em relação ao conceito e ao tipo de força. I II 0 0 As forças de ação e reação sempre atuam em corpos distintos. 1 1 A força elástica é proporcional à deformação da mola. 2 2 A força normal é uma força de reação ao peso. 3 3 Força é uma grandeza vetorial. 4 4 Uma força sempre causa mudança no valor da velocidade.

119. Um carrinho de montanha-russa, de massa igual a 20kg, parte do repouso no ponto A, a uma altura HÁ = 50m e, após percorrer o trilho indicado na figura, alcança a altura máxima HC = 45m, no ponto C, de onde retorna. Analise as proposições apresentadas.

I II

0 0 A variação da energia potencial do carrinho entre os pontos A e C é igual a 1000 joules.

1 1 O carrinho tem a mesma velocidade, ao passar pelo ponto B, na subida e na descida.

2 2 Se o percurso total AC tem 125m de comprimento, a força média de atrito que atuou sobre o carrinho foi igual a 8 N.

3 3 A soma das energias cinética e potencial gravitacional é sempre decrescente ao longo do caminho AC.

4 4 Ao retornar, o carrinho não conseguirá atingir o ponto A.

h = 1,2 m m

k

0,2 m

A

k

B

F(N)

0

50

7000

x(m)

HA HC

C

A

B

FÍSICA – MECÂNICA

AUTORIA – PROF. MARCELO CORREIA E-mail: [email protected] 15

DINÂMICA – PARTE 3 Nesta terceira parte do estudo da dinâmica vamos estudar grandezas físicas muito importantes: quantidade de movimento (que também pode ser chamada de momento linear) e impulso. Vamos discutir um princípio de conservação muito importante: O princípio de conservação do momento linear. Nesta parte vamos fazer uma introdução para o estudo de um sistema de partículas, na verdade, iremos ver como calcular o momento linear de um sistema de partículas.

MOMENTO LINEAR OU QUANTIDADE DE MOVIMENTO Definimos a quantidade de movimento (ou momento linear) de uma partícula como sendo o produto da massa “m” da partícula pela sua velocidade vetorial “ v

r”. Assim temos:

Onde:

Qr

���� Momento linear ou quantidade de movimento m ���� massa da partícula vmQ

rr

⋅=

vr

���� Velocidade vetorial Nota:

• Observe que o momento linear e a velocidade têm a mesma direção e sentido;

• A unidade, no SI, do momento linear é: kg� m/s.

IMPULSO

Definimos o impulso como sendo o produto da força “Fr

” que atua sobre a partícula pelo intervalo de tempo “∆t” que esta força atua. Assim temos:

Onde:

Ir

���� Impulso

Fr

���� força tF ∆⋅=rr

I ∆t ���� intervalo de tempo que a força F

r

atua

Nota: • Observe que o impulso e a força têm a mesma direção e sentido; • A unidade, no SI, do momento linear é: N� s. • Podemos constatar facilmente que a unidade de impulso no SI é

equivalente a unidade de momento linear no SI, isto é: N� s = kg� m/s. • A observação anterior mostra que existe uma ligação entre as

grandezas vetoriais momento linear e impulso, esta ligação é mostrada e expressa em forma de um teorema chamado TEOREMA DO IMPULSO.

Teorema do Impulso

O teorema do impulso pode ser enunciado da seguinte forma:

O IMPULSO DA FORÇA RESULTANTE QUE ATUA SOBRE UMA PARTÍCULA É IGUAL À VARIAÇÃO DA SUA QUANTIDADE DE

MOVIMENTO (MOMENTO LINEAR).

Assim temos:

Qrr

∆=I

Gráfico da Força em Função do Tempo No gráfico da força em função do tempo o impulso é dado (numericamente) pela área limitada pelo gráfico e o eixo dos tempos.

Cálculo do Momento Linear de um Sistema de Partículas Um sistema de partículas nada mais é do que um conjunto de partículas. Considere um conjunto de “n” partículas. Cada partícula tem a sua massa e sua velocidade vetorial, assim:

• A partícula 1 tem massa m1 e velocidade vetorial 1vr

;

• A partícula 2 tem massa m2 e velocidade vetorial 2vr

;

• A partícula 3 tem massa m3 e velocidade vetorial 3vr

;

• A partícula n tem massa mn e velocidade vetorial nvr

;

Assim, cada partícula terá o seu momento linear:

• A partícula 1 tem momento linear 111 vmQrr

⋅= ;

• A partícula 2 tem momento linear 222 vmQrr

⋅= ;

• A partícula 3 tem momento linear 333 vmQrr

⋅= ;

• A partícula n tem momento linear nnn vmQrr

⋅= .

O momento linear do sistema de partículas “ Qr

” é dado pela soma dos momento lineares das n partículas que constituem o sistema. Assim temos:

∑=

=n

i 1iQQrr

ou n21 QQQQr

L

rrr

+++=

Conservação do Momento Linear (Quantidade de Movimento)

Podemos enunciar o princípio de conservação do momento linear da seguinte forma:

NUM SISTEMA DE PARTÍCULAS EM QUE NÃO ATUA AGENTE EXTERNO O MOMENTO LINEAR É CONSERVADO, ISTO É, É

CONSTANTE.

Assim podemos escrever que: SE NÃO ATUA AGENTE EXTERNO

finalinicial QQrr

=

Colisões ou Choques

As colisões ou choques são estudados tomando a conservação do momento linear, por isso é pertinente que fazer referência ao assunto. Podemos considerar três tipos de colisões, que são:

• Colisão Perfeitamente Elástica ���� Quando após a colisão os corpos se separam e não ficam com deformações. Nesta colisão há conservação do momento linear e da energia cinética do sistema de partículas que colide entre si;

• Colisão Parcialmente Elástica ���� Quando após a colisão os corpos se separam e pelo menos um dos corpos permanece deformado. Nesta colisão há conservação do momento linear e há dissipação de energia cinética, isto é, a energia cinética depois da colisão é menor do que a energia cinética antes da colisão;

• Colisão Perfeitamente Inelástica ���� Quando após a colisão os corpos se deslocam juntos. Nesta colisão há conservação do momento linear e a há uma dissipação máxima de energia cinética em relação aos outros dois tipos de colisões.

Nota: Observe que em todos os tipos de colisão há conservação do momento linear.

A

F

t

0

����

An

=I x

FÍSICA – MECÂNICA

AUTORIA – PROF. MARCELO CORREIA E-mail: [email protected] 16

BRINQUEDINHO DE VESTIBULANDO!!!!!

120. Um carrinho de supermercado, carregando latas de óleo, foi abandonado numa rampa e adquiriu uma velocidade escalar constante de 50cm/s. Sabendo que sua massa total é de 20kg, qual o módulo da quantidade de movimento do carrinho, em kg� ms?

(a) 1 (b) 12 (c) 15 (d) 10 (e) 20

121. Duas partículas 1 e 2 têm massas iguais, e suas respectivas energias cinéticas Ec1 e Ec2 são tais que Ec1 = 4Ec2. Calcule a razão entre os módulos de suas quantidades de movimento.

122. Uma partícula de massa m = 10kg realiza um MCU com velocidade de módulo 1,0m/s. Determine o módulo da variação de seu momento linear nos seguintes intervalos de tempo: I. Um quarto de período; II. Meio período; III. Três quartos de período; IV. Um período.

123. (FCMSC–SP) Em uma carta de Benjamin Franklin, como objeto à teoria corpuscular da luz, ele declarava: “Uma partícula de luz, caminhando com a velocidade da luz, deveria produzir o mesmo impacto que uma bala de canhão e massa 10kg animada de velocidade de 300m/s, ao atingir a superfície da Terra”. Nestas condições, a partícula de luz a que se referia Franklin deveria ter massa, expressa em kg, de ordem de grandeza igual a:

(a) 10–8 (b) 10–6 (c) 10–5 (d) 10–7 (e) 10–4

124. Num jogo de vôlei, um jogador, ao efetuar uma cortada, aplicada na bola um impulso de intensidade de 80N� s. Se a duração da cortada é de 0,20s, determine a intensidade média da força que o jogador aplica na bola.

BRINCANDO COM A COVEST (UFPE – UFRPE)

125. Uma criança de massa igual a 30kg e um homem de massa igual a 60kg estão parados em pé, um em frente ao outro, numa pista de patins. De repente, o homem empurra a criança para trás com uma velocidade de 1m/s. Pode-se afirmar que a velocidade de recuo do homem será, em m/s, igual a:

(a) 0,5 (b) 0,8 (c) 1,0 (d) 1,5 (e) 2,0

126. Um menino, sentado numa canoa parada na superfície de um lago, atira um tijolo par fora. A massa do menino e da canoa é, no total, 40kg. Sabendo que a massa do tijolo é 0,4kg, e que sua velocidade, ao sair da mão do menino, é de 10m/s em relação à água, qual é a velocidade, em cm/s, com que a canoa começa a se movimentar?

(a) 12 (b) 15 (c) 10 (d) 8 (e) 20

127. Um vagão corre ao longo de trilhos horizontais, sem atrito. Em determinado instante, colide com outro, de igual massa, que estava inicialmente parado. Os dois vagões passam então a se mover unidos. Se a energia cinética inicial do primeiro vagão era igual a 40kJ, quanto deve valer, em kJ, a energia cinética dos dois vagões após a colisão?

128. (Modificada) Durante o jogo de futebol, professores versus alunos do MEGA, o professor Eriberto é atingido frontalmente por uma bola de massa 500g, a uma velocidade de 72km/h. Qual a força média, em 103N, que atua no rosto do professor Eriberto durante o impacto, se a bola retorna no sentido oposto com a mesma velocidade em módulo, e a colisão teve a duração de 0,01s?

129. Uma força aplicada durante 1s a um objeto de massa 10kg varia de intensidade conforme o gráfico abaixo. Qual o impulso total da força, em N�s, após a interação?

(a) 10 (b) 15 (c) 20 (d) 25 (e) 30

130. O gráfico abaixo representa a variação da velocidade com o tempo de um objeto de massa igual a 10kg que se desloca em linha reta. Qual a variação do momento linear do objeto, em kg� m/s, a cada intervalo de 2s?

(a) 20 (b) 40 (c) 60 (d) 80 (e) 100

131. Um jogador de tênis pode sacar a bola com velocidade de 50m/s. Sabendo que a massa de uma bola de tênis é 60g, calcule o impulso (em unidades do sistema MKS) fornecido a bola quando ela é sacada.

132. A figura mostra a variação no tempo da intensidade de uma força F que atua sobre um corpo de massa igual a 2,5kg. Sabendo que em t = 0 o corpo estava em repouso, e que a força tem direção constante, calcule a velocidade do corpo no instante t = 10s.

133. Um átomo de argônio, movendo-se com velocidade igual a 400m/s, choca-se elasticamente contra a parede de um recipiente. Se a massa do átomo de argônio é de 6,5� 10–26kg, qual o impulso sobre a parede, durante

o impacto, em unidades de 10–24N?

134. Um patinador de 65 kg, em repouso, arremessa um peso de 5,0 kg, horizontalmente para frente. A velocidade do peso em relação ao patinador é de 3,5 m/s no instante do arremesso. Calcule o módulo da velocidade em relação à Terra, adquirida pelo patinador, em cm/s. Despreze o atrito entre os patins e o piso.

135. Um rapaz de 59 kg está parado sobre um par de patins, no instante em que ele pega um pacote de 1,0 kg que foi jogado em sua direção. Depois de apanhar o pacote, o rapaz recua com uma velocidade igual a 0,3 m/s. Qual a velocidade horizontal do pacote, em m/s, imediatamente antes de ele ser apanhado? Despreze o pequeno atrito do solo com as rodas dos patins.

136. Um bloco de massa m1 = 100 g comprime uma mola de constante elástica k = 360 N/m, por uma distância x = 10,0 cm, como mostra a figura. Em um dado instante, esse bloco é liberado, vindo a colidir em seguida com um outro bloco de massa m2 = 200 g, inicialmente em repouso. Despreze o atrito entre os blocos e o piso. Considerando a colisão perfeitamente inelástica, determine a velocidade final dos blocos, em m/s.

0,5 1,0 0

10

20 F(N)

t(s)

1,0

2,0

0

2 1

F(N)

t(s)

3

4

5

6

7

10,0

2,0

0

6,0

F(N)

t(s)

4,0

20

–10

8,0

V v E

m1

k

m2

10 cm

FÍSICA – MECÂNICA

AUTORIA – PROF. MARCELO CORREIA E-mail: [email protected] 17

DINÂMICA – PARTE 4 Nesta quarta parte do estudo da dinâmica vamos estudar sistemas de partículas e corpo rígido: centro de massa de um sistema de partículas será o nosso primeiro objeto de estudo seguido das grandezas dinâmicas que estão envolvidas no estudo da rotação de um corpo rígido (energia cinética rotacional, momento de inércia, momento angular e conservação do momento angular). Esta parte da dinâmica é muito importante para a 2ª fase da Covest e principalmente para UPE.

CENTRO DE MASSA Um sistema mecânico pode ser formado por um número muito grande de partículas, é o que chamamos de sistema de partículas. O que chamamos de corpo rígido é nada mais do que um conjunto infinito de partículas, tal como uma cadeira. O movimento das diversas partículas que constituem o nosso sistema, em diversos casos, poderá ser estudado considerando-se o movimento de um único ponto em que consideramos que a massa de todas as partículas está concentrada neste ponto. A este único ponto chamamos de centro de massa. Para localizar o centro de massa devemos adotar um sistema de referência (sistema cartesiano) e aplicar as equações mostradas a seguir:

=

=

⋅=

n

i

n

i

1

1

i

ii

CM

m

xmx ou

n21

nn2211CM mmm

xmxmxmx

+++⋅++⋅+⋅=

L

L

=

=

⋅=

n

i

n

i

1

1

i

ii

CM

m

ymy

ou

n21

nn2211CM mmm

ymymymy

+++⋅++⋅+⋅

=L

L

=

=

⋅=

n

i

n

i

1

1

i

ii

CM

m

zmz

ou n21

nn2211CM mmm

zmzmzmz

+++⋅++⋅+⋅

=L

L

A figura abaixo ilustra a aplicabilidade do que foi exposto anteriormente:

BRINCANDO COM A COVEST (UFPE – UFRPE)

137. A figura representa a molécula de NO. O átomo de nitrogênio tem massa atômica 14 e o átomo de oxigênio tem massa atômica 16. A distância entre os dois átomos é D = 1,5� 10–8cm. Qual a distância entre o centro de massa da molécula e o átomo de nitrogênio, em unidades de 10–10cm?

138. A figura mostra uma estrutura vertical formada por três barras iguais, homogêneas e de espessuras desprezíveis. Se o comprimento de cada barra é 90cm, determine a altura, em cm, do centro de massa do sistema, em relação ao solo.

139. Uma chapa de aço, uniforme, é cortada na forma indicada na figura abaixo. À que distância do ponto P, em cm, está localizado o centro de massa da chapa?

140. Duas partículas, de massas M1 = M e M2 = M/2, estão presas por uma haste de comprimento L = 48cm e massa desprezível, conforme a figura. Qual à distância, em centímetros, do centro de massa do sistema em relação à posição da partícula de massa M1?

141. Uma caixa cúbica, sem tampa, com 40cm de lado, é feita de chapas de metal homogêneas de espessura desprezível. Determine a localização do centro de massa da caixa em relação ao fundo (com tampa) e duas de suas faces laterais.

142. A figura mostra as dimensões de uma placa composta; metade da placa é feita de alumínio (densidade = 2,70g/cm3) e metade ferro (densidade = 7,85g/cm3). Qual a localização do centro de massa da placa em relação ao seu centro geométrico?

x1

x2

x3

z2

zn

z1

y2

y1

xn

y3

z3

yn

m1

m2

m3

mn

y

z

x

D

N O

90cm

218

218 29

29

M1 M2

L

Alumínio Ferro

12,0cm

24,0cm

12,0cm 12,0cm

2,0cm

FÍSICA – MECÂNICA

AUTORIA – PROF. MARCELO CORREIA E-mail: [email protected] 18

ROTAÇÃO DO CORPO RÍGIDO É importante lembrar das grandezas cinemáticas que descrevem o movimento circular, portanto você pode fazer uma revisão nas páginas 26 e 27 na parte de cinemática para recordar estas grandezas. Um corpo rígido em rotação pura é um conjunto de partículas descrevendo movimento circular e sendo assim as grandezas angulares são mais apropriadas para o seu estudo. Sabendo que já foram discutidas as grandezas cinemáticas que precisamos para entender a rotação, que são basicamente: posição angular, velocidade angular, aceleração angular e aceleração vetorial (tangencial e centrípeta), vamos nos preocupar com as grandezas dinâmicas.

ENERGIA CINÉTICA ROTACIONAL Um corpo em rotação tem energia cinética, cada partícula que constitui o corpo tem energia cinética que é dada pela nossa conhecida expressão: EC = ½ m� v2. Assim, a energia cinética do corpo é a soma das energias cinéticas de todas as partículas que constituem o corpo, logo, temos:

2

vm

2

vm

2

vmE

2nn

222

211

CRot

⋅++⋅+⋅= L

O problema na expressão acima é a de que cada partícula tem uma velocidade escalar diferente dificultando o cálculo da energia cinética. Podemos, no entanto, resolver este problema se substituirmos a velocidade escalar de cada partícula pela sua corresponde velocidade angular, pois sabemos que todas as partículas do corpo em rotação pura têm a mesma velocidade angular, isto pode ser feito lembrando que: v = ωωωω� R, onde: ω � velocidade angular e R � é o raio da trajetória descrita pela partícula (é a distância entre a partícula e o eixo de rotação do corpo rígido). Fazendo a substituição, temos:

( ) ( ) ( )2

ωrm

2

ωrm

2

ωrmE

2nn

222

211

CRot

⋅++⋅+⋅= L

( )2nn

222

211

2CRot rmrmrmω

2

1E ⋅++⋅+⋅⋅= L

( )∑=

⋅⋅=n

1i

2ii

2CRot rmω

2

1E

Na expressão acima destacamos o fator: ( )∑=

⋅=n

1i

2ii rmI que é

chamado de MOMENTO DE INÉRCIA ou ENÉRCIA ROTACIONAL do corpo em relação ao eixo de rotação. Substituindo na equação da energia cinética que encontramos, teremos finalmente:

2CRot ω

2

1E ⋅= I

Observe que a inércia rotacional desempenha um papel na rotação análogo ao papel desempenhado pela massa na translação. Observe que a expressão para a energia cinética de um corpo é dada pela soma de sua energia cinética de translação com a sua energia cinética de rotação, assim:

CRotCTransCTotal EEE +=

22CTotal ω

2

1v

2

1E ⋅+⋅= Im

BRINQUEDINHO DE VESTIBULANDO!!!!!

143. Calcule o momento de inércia de uma roda que tem 24400J de energia

cinética de rotação ao girar a 600rpm. Considere π = 3,14.

144. Um disco de 8kg raio igual a 0,5m gira com uma velocidade na periferia de 4m/s. Sabendo que o seu momento de inércia é dado por:

2rm

2

1⋅⋅=I , encontre a sua energia cinética rotacional.

145. Considerando o sistema de partículas da figura encontre: I. O momento de inércia do sistema de partículas em relação ao

eixo dos y que passa pelas partículas m3 e m4; II. O momento de inércia do sistema de partículas em relação ao

eixo z que passa por m1 e m4; III. O momento de inércia do sistema de partículas em relação ao

eixo dos x que passa por m4 e é perpendicular ao plano do quadrado.

146. A figura a seguir mostra um sistema constituído por oito partículas idênticas, todas de massa m = 4 kg. Considerando que as barras que ligam as partículas tem massas desprezíveis encontre: I. Inércia rotacional do sistema de partículas em relação ao eixo dos

x; II. A inércia rotacional do sistema de partículas em relação ao eixo

dos y; III. A inércia rotacional do sistema de partículas em relação ao eixo

dos z;

147. Com relação a questão anterior, considere que o sistema de partícula está em rotação cujo eixo de rotação é o eixo dos x. Sabendo que a velocidade angular do sistema de partículas vale 8 rad/s calcule sua energia cinética rotacional.

148. Com relação a questão 146, considere que o sistema de partícula está em rotação cujo eixo de rotação é o eixo dos y. Sabendo que a velocidade angular do sistema de partículas vale 8 rad/s calcule sua energia cinética rotacional.

149. Com relação a questão 146, considere que o sistema de partícula está em rotação cujo eixo de rotação é o eixo dos z. Sabendo que a velocidade angular do sistema de partículas vale 8 rad/s calcule sua energia cinética rotacional.

m1 m2

m3 m4

x

y

z

2,0m

2,0m

m1 = 3 kg m2 = 4 kg m3 = 3 kg m4 = 4 kg

8m

6m

2m

x

z

y

FÍSICA – MECÂNICA

AUTORIA – PROF. MARCELO CORREIA E-mail: [email protected] 19

TORQUE OU MOMENTO DE UMA FORÇA Para provocar rotação em um corpo, a força aplicada não depende apenas da intensidade ser suficiente, mas de onde é aplicada. Considere o corpo, na figura abaixo, e observe que as forças F1 e F2 podem provocar rotação em torno do eixo de rotação 0, porém as forças F3 e F4, por mais intensas que sejam, não provocam rotação. Na figura anterior os vetores r1, r2, r3 (não aparece porque seu módulo é zero) e r4 são chamados de braço e indicam onde as forças estão sendo aplicadas em relação ao ponto 0 (eixo de rotação). Observe que a força F3 não provoca rotação, isto é, não provoca torque porque está aplicada no próprio eixo de rotação: imagine que você vai abrir uma porta e aplica a força onde está a dobradiça. Já a força F4 não provoca rotação porque está aplicada na mesma direção do braço e sendo assim não provoca torque: imagine que você vai abrir uma porta e aplica a força ao longo da direção da porta. Na figura seguinte nos concentramos numa força e observamos que a componente da força que atua no corpo que provoca torque é àquela que é perpendicular ao braço, que representamos por F┴. A componente da força tangencial, representada por Ftg, ao braço não contribui para a produção de torque. Observando que F┴ = F� senθ e que o torque é proporcional a esta componente da força e ao braço, podemos definir o torque ou momento de uma força como sendo um vetor dado pelo produto vetorial entre o braço e a força, assim temos:

FrMrrr

×= Como já sabemos do produto vetorial temos que:

• Módulo: M = r� F� senθ, onde θ é o ângulo formado entre o braço e a força, r é o módulo do braço e F é o módulo da força;

• Direção e sentido são dados pela regra da mão esquerda que é lembrada na figura a seguir.

Escrevendo a 2ª Lei de Newton para a Rotação Não é difícil perceber que o torque desempenha na rotação o mesmo papel que a força representa na translação, portanto podemos escrever a segunda lei de Newton em termos de torque para a rotação. Assim podemos mostrar que:

α⋅=IresM

Na expressão acima observamos que o módulo do torque resultante é o momento de inércia ou inércia rotacional (análogo da massa para a rotação) multiplicado pela aceleração angular (aceleração adequada para descrever a variação do momento para a rotação). Notas:

• A unidade de torque no sistema internacional é o N� m que não deve ser confundido com a unidade de trabalho N� m = J (joule). Pois estas duas grandezas e suas unidades são completamente diferentes;

• O torque resultante é a soma vetorial de todos os torques que atuam no corpo. Cada força que a tua no corpo provoca um torque (incluindo a possibilidade de ser nulo), assim, a soma vetorial destes torques é o torque resultante, tal como a soma de todas as forças que atuam num corpo é a força resultante.

MOMENTO ANGULAR OU QUANTIDADE DE MOVIMENTO ANGULAR

O momento angular é o análogo rotacional do momento linear (quantidade de movimento). O momento angular de uma partícula é definido como sendo o produto vetorial da posição da partícula em relação ao centro de curvatura da trajetória pelo momento linear da partícula, assim temos:

QrLrrr

×= Como já sabemos do produto vetorial temos que:

• Módulo: L = r� Q� senθ, onde θ é o ângulo formado entre a posição em relação ao centro de curvatura da trajetória e o momento linear, r é o módulo da posição e Q é o módulo do momento linear;

• Direção e sentido são dados pela regra da mão esquerda que é lembrada na figura a seguir;

• Lembre que o momento linear (ou quantidade de movimento) é a

grandeza vetorial definida como: vmQrr

⋅= ; • O momento angular de um sistema de partículas é a soma vetorial do

momento angular de todas as partículas que constituem o sistema; • A unidade de momento angular no SI é o kg� m2/s = J� s;

Momento Angular de um Corpo Rígido

Não é difícil mostrar que para um corpo rígido o momento angular é dado por:

ω⋅=IL Isto é, o momento angular é o produto do momento de inércia (ou inércia rotacional) do corpo pela sua velocidade angular.

Sabemos que a taxa de variação do mento linear em relação ao tempo é igual a força resultante que atua no corpo, analogamente temos que a taxa de variação temporal do momento angular é igual ao torque resultante, assim podemos escrever que:

tL

MRes ∆∆=r

r

Eixo de rotação

Ftg

r

θ

F

F┴

0

Eixo de rotação

F2

F4

F1 F3

r1

r4 r2

θ2

θ1

0

FrMrrr

×= Fr

rr

FÍSICA – MECÂNICA

AUTORIA – PROF. MARCELO CORREIA E-mail: [email protected] 20

Conservação do Momento Angular Podemos enunciar o princípio de conservação do momento angular da seguinte forma:

NUM SISTEMA DE PARTÍCULAS EM QUE NÃO ATUA TORQUE

EXTERNO (RESULTANTE) O MOMENTO ANGULAR É CONSERVADO, ISTO É, É CONSTANTE.

Assim podemos escrever que:

SE NÃO ATUA AGENTE EXTERNO

finalinicial LLrr

=

BRINQUEDINHO DE VESTIBULANDO!!!!!

150. Um tubo de paredes finas rola pelo chão. Seu momento de inércia em relação a um eixo paralelo ao seu comprimento e que passa pelo seu centro de massa é dado por: I = m� R2, em que R é seu raio e m é sua massa. Encontre a razão entre as sus energias cinética translacional e rotacional, em torno de um eixo paralelo ao seu comprimento e que passa pelo seu centro de massa.

151. Uma pequena (tamanho desprezível) bola de 4kg está presa a uma das extremidades de uma haste de 2m de extensão e de massa desprezível, a outra extremidade está presa a um eixo no teto, formando um pêndulo. Quando o pêndulo é desviado 30º da vertical, qual o módulo do torque em relação ao eixo?

152. Uma pequena (tamanho desprezível) bola de 4,0kg está presa a uma das extremidades de uma haste de 2,0m de extensão e de massa igual a 1,0kg, a outra extremidade está presa a um eixo no teto, formando um pêndulo. Quando o pêndulo é desviado 30º da vertical, qual o módulo do torque em relação ao eixo?

153. Um ciclista de 75kg cola todo o peso de seu corpo em cada um dos pedais, para movimentá-los para baixo. Considerando que a roda da corrente, onde o pedal está preso, tem 40cm de diâmetro e que o pedal é instalado de forma a ficar rente a periferia da roda da corrente, determine o módulo do torque máximo que o ciclista pode exercer nesse processo.

154. Uma roda tem uma aceleração angular de 20rad/s2, quando o torque 300N� m é aplicado sobre ela. Qual o momento de inércia da roda?

155. A figura abaixo mostra dois blocos de massas iguais suspensos nas extremidades de uma haste rígida, sem peso, de comprimento L1 = 20cm e L2 = 80cm. A haste é mantida na horizontal e então é solta. Calcule a aceleração dos dois blocos quando eles começam a se mover.

156. Uma chaminé alta, de forma cilíndrica, cai se houver uma ruptura na sua base. Tratando a chaminé como um bastão fino, de altura h, encontre a componente tangencial da aceleração linear do topo da chaminé quando esta formar 30º com a vertical. Sabe-se que o momento de inércia de um bastão em relação a um eixo de rotação na extremidade é dado por: I = (m� L2)/3, onde: m é a massa do bastão e L é o seu comprimento.

157. Um projétil de massa m = 2kg é atirado do chão obliquamente formando um ângulo de 60º com a horizontal. Sabendo que a velocidade inicial de lançamento é 10m/s e a aceleração da gravidade local 10m/s2, encontre o momento angular, em unidades do SI, do projétil 2s após o seu lançamento.

158. O rotor de um motor elétrico tem inércia rotacional de 2,0� 10–3kg� m2 em torno do seu eixo central. O motor é instalado para mudar a orientação de uma sonda espacial. O eixo do rotor é instalado de forma paralela ao eixo da sonda, que tem momento de inércia igual a 24kg� m2 em torno do seu eixo. Encontre o número de revoluções que tem que dar o rotor para fazer a sonda girar um ângulo de 30º em torno do seu eixo.

159. (UPE–2004) Um menino está sentado em uma cadeira que está girando em torno de um eixo vertical, com velocidade angular ù0, conforme figura. O menino tem os braços estendidos e segura um altere em cada mão, de modo que o momento de inércia do sistema (menino, halteres e assento) é I0 . O menino abraça rapidamente os halteres, de modo que o momento de inércia final do sistema reduza de 70% do momento de inércia inicial. Desprezando o torque devido ao atrito no eixo da cadeira, durante o intervalo de tempo no qual o momento de inércia do sistema varia, é correto

afirmar que a velocidade angular final do sistema é aproximadamente: (a) ù0 (b) 3,33� ù0 (c) 1,43� ù0 (d) 0,3� ù0 (e) 0,7� ù0

160. Suponha que o combustível do Sol se extinga e ele, subitamente, entre em colapso formando um tipo de estrela denominada anã branca, com um diâmetro igual ao da Terra. Considerando que não houvesse perda de massa e que o Sol é uma esfera maciça e homogênea, encontre o novo período do Sol nestas condições. Dados: Período do Sol: 25 dias; Massa do Sol: ≈ 2� 1030kg; Diâmetro do Sol: ≈14� 108m; massa da Terra: ≈ 6� 1024kg; diâmetro da Terra: ≈ 12� 106m; momento de inércia de uma esfera maciça em relação a um eixo de rotação em que seu diâmetro está contido é dado por: I = (2� m� R2)/5, onde: m é a massa da esfera e R é o seu raio.

161. O momento de inércia de uma estrela girando (considere com uma esfera maciça e homogênea) que está em colapso cai a um terço do seu valor inicial. Qual o fator de aumento de sua energia cinética rotacional.

ESTÁTICA

Trataremos a partir de agora da estática. A estática é a parta da mecânica que estuda os corpos em equilíbrio estático e na verdade é um caso particular da dinâmica. A estática pode ser dividida em três partes:

• Estática do ponto material; • Estática do corpo rígido ou do corpo extenso; • Estática dos fluidos ou Fluidostática ou ainda Hidrostática.

Estática do Ponto Material

A estática do ponto material trata do equilíbrio estático das partículas, isto é, dos corpos que podemos desprezar suas dimensões na análise do fenômeno. A estática do ponto material é basicamente aplicar a condição de equilíbrio de translação, já que não tem sentido tratar de rotação de uma partícula. Já foi discutida a condição de equilíbrio de translação que:

SE UMA PARTÍCULA ESTÁ EM EQUILÍBRIO ESTÁTICO A FORÇA RESULTANTE QUE ATUA SOBRE A MESMA É NULA. O que devemos fazer é simplesmente sofisticar esta condição no

sentido de aplicá-la de uma forma mais sistemática. Isto é feito mediante a adoção de um sistema de referência retangular (sistema cartesianos) o que nos faz poder escrever a condição de equilíbrio de translação da seguinte forma:

20cm 80cm

FÍSICA – MECÂNICA

AUTORIA – PROF. MARCELO CORREIA E-mail: [email protected] 21

SE UMA PARTÍCULA ESTÁ EM EQUILÍBRIO ESTÁTICO:

=

=⇒=

∑∑

0F

0F0F

y

x

R r

r

r

Estática do Corpo Extenso

A estática do corpo extenso trata do equilíbrio estático dos corpos extensos, isto é, dos corpos que não podemos desprezar suas dimensões na análise do fenômeno. A estática do corpo extenso é basicamente aplicar a condição de equilíbrio de translação e a condição de equilíbrio de rotação, já que os corpos extensos podem viver movimento de rotação e/ou translação: A condição de translação para o corpo extenso a mesma condição aplicada para a partícula. SE UM CORPO EXTENSO ESTÁ EM EQUILÍBRIO ESTÁTICO A FORÇA

RESULTANTE QUE ATUA SOBRE O MESMO É NULA. O que adotando um sistema de referência da mesma forma descrita

para as partículas leva-nos à mesma equação:

SE UM CORPO EXTENSO ESTÁ EM EQUILÍBRIO ESTÁTICO:

=

=⇒=

∑∑

0F

0F0F

y

x

R r

r

r

A condição de equilíbrio de rotação pode ser enunciada da seguinte forma:

SE UM CORPO EXTENSO ESTÁ EM EQUILÍBRIO ESTÁTICO O TORQUE RESULTANTE QUE ATUA SOBRE O MESMO É NULO.

∑ =⇒= 0M0M R

r

Na equação acima para facilitar o cálculo do torque calculamos seu módulo e adotamos a seguinte convenção para efetuar o somatório:

• Se a força que provoca o torque tende a rotar o corpo no sentido anti-horário o torque é positivo;

• Se a força que provoca o torque tende a rotar o corpo no sentido horário o torque é negativo.

Estabilidade do Equilíbrio

Podemos classificar o equilíbrio estático como sendo: • Equilíbrio indiferente ou neutro ���� É aquele que ocorre quando

forças e/ou torques não existiram após a perturbação do equilíbrio no sentido de restaurar o afastar o corpo da posição de equilíbrio.

• Equilíbrio instável ���� É aquele que ocorre quando forças e/ou torques provocadas por um pequeno deslocamento do corpo em relação a posição de equilíbrio provocam um afastamento ainda maior desta posição de equilíbrio. Isto é, se uma força e/ou torque deslocar o corpo da posição de equilíbrio retira-o desta posição o afastando mais ainda da antiga posição de equilíbrio.

• Equilíbrio estável ���� É aquele que ocorre quando forças e/ou torques provocadas por um pequeno deslocamento do corpo atuam de modo que fazem o corpo retornar à posição de equilíbrio. Isto é, o corpo é perturbado da posição de equilíbrio mais retorna a esta após a perturbação. Em resumo podemos dizer que: se um sistema for ligeiramente

perturbado na sua posição de equilíbrio, o equilíbrio será estável se o sistema retornar espontaneamente para a sua posição de equilíbrio original, será instável se o sistema se afastar espontaneamente da posição de equilíbrio original e será indiferente (ou neutro) se nem forças e/ou torques atuarem para modificar a posição perturbada do sistema.

BRINQUEDINHO DE VESTIBULANDO!!!!!

162. (COVEST) Uma pessoa usa uma corda para atravessar um rio conforme a figura. A corda foi amarrada em cada uma das extremidades de modo a ficar aproximadamente horizontal se ninguém estiver pendurado nela. Quando pessoa atinge o meio da travessia a tensão na corda é, em módulo, igual ao seu peso. Qual será o ângulo θ, em graus, nesta situação?

163. (COVEST) Duas partículas idênticas, de massa m = 0,9kg cada, estão em repouso. Suspensas pro fios inextensíveis como mostrado na figura abaixo. Os fios A e B estão presos no teto e fazem ângulos de 45º com a vertical. A tração no fio horizontal C vale:

(a) 2,0N (b) 4,0N (c) 6,0N (d) 8,0N (e) 9,0N

164. (COVEST) A figura mostra um peso de 44 N suspenso no ponto P de uma corda. Os trechos AP e BP da corda formam um ângulo de 90o, e o ângulo entre BP e o teto é igual a 60o. Qual é o valor, em newtons, da tração no trecho AP da corda?

165. (COVEST) A figura abaixo mostra um dispositivo constituído de um suporte sobre o qual uma trave é apoiada. Na extremidade A, é suspenso um objeto, de massa 95 kg, enquanto se aplica uma força vertical F na extremidade B, de modo a equilibrar o objeto. Desprezando o peso da trave, em relação ao peso do objeto, calcule o módulo da força F necessária para equilibrar o objeto, em N.

166. (COVEST) Uma barra horizontal de massa desprezível possui uma de

suas extremidades articulada em uma parede vertical. A outra extremidade está presa à parede por um fio que faz um ângulo de 45º com a horizontal e possui um corpo de 55 N pendurado. Qual o módulo da força normal à parede, em newtons, que a articulação exerce sobre a barra?

A

B

0,5 m

5m

trave

suporte

θ

θ

m

m

45º

45º

A

C

B

45º

fio

60o

B

A

P

FÍSICA – MECÂNICA

AUTORIA – PROF. MARCELO CORREIA E-mail: [email protected] 22

167. (COVEST) A figura mostra uma barra homogênea, de comprimento L=1,0m, presa ao teto nos pontos A e B por molas ideais, iguais de constante elástica k=1,0××××102 N/m. A que distância do centro da barra, em centímetros, deve ser pendurado um jarro de massa m=2,0 kg, de modo que a barra permaneça na horizontal?

168. (COVEST) A escada AB está apoiada numa parede sem atrito, no ponto B, e encontra-se na iminência de escorregar. O coeficiente de atrito estático entre a escada e o piso é 0,25. Se a distância de A até o ponto O é igual a 45 cm, qual a distância de B até O, em centímetros?

169. (COVEST) Deseja-se saber a massa de uma régua de 1,0 m de

comprimento e dispõe-se de um pequeno corpo de 9,0 g. Realiza-se o experimento mostrado abaixo. Apóia-se a régua, na iminência de cair, sobre a borda de uma mesa horizontal, com o corpo na extremidade da régua (ver figura). O ponto P coincide com a marcação 45 cm e alinha-se com a borda da mesa. O ponto Q indica o ponto médio da régua e o pequeno corpo coincide com a marcação 0,0 cm. Calcule a massa da régua, em g.

170. (COVEST) A gangorra da figura abaixo está equilibrada em torno do ponto C por efeito das massas mA = 20kg e mB = 40kg. Indique o comprimento total AB, em metros, supondo que AC = 6,0m. Despreze a massa da gangorra.

(a) 7,0 (b) 7,5 (c) 8,0 (d) 8,5 (e) 9,0

171. (COVEST) Uma tábua uniforme de peso igual a 48N e 3,6m de comprimento repousa horizontalmente sobre dois cavaletes, conforme a figura. Qual a força normal, em N, exercida sobre a tabula no ponto P?

FLUIDOSTÁTICA (HIDROSTÁTICA) Vamos agora discutir a ultima parte da estática. A Fluidostática, comumente chamada de Hidrostática, estuda os fluidos que estão em equilíbrio estático, isto é, que estão em repouso.

Densidade e Massa Específica A densidade de um corpo será representada pela letra grega ρ (rô) e é definida como sendo a razão entre a massa “m” do corpo e o seu volume “V”. Já a massa específica de uma substância representaremos pela letra grega µ (mi) e é definida como a massa “m” de uma determinada porção da substância e o volume “V” ocupado por esta porção da substância. Observe que ambas as grandezas: densidade e massa específica são razões da massa pelo volume. A diferença entre elas é que a densidade é para corpos e a massa específica e par a substância. Por exemplo: Um bloco de ferro é um corpo e não precisa necessariamente ter densidade igual a massa específica do ferro, isto só ocorre quando o corpo for homogêneo e maciço o que será considerado na maioria dos casos, principalmente no nosso caso que estamos estando fluidos. Assim, se o corpo for homogêneo e maciço a densidade e massa específica se tornam iguais. Matematicamente temos:

Densidade ����V

mρ= Massa Específica ����

V

mµ=

A unidade de densidade ou massa específica no SI é o: kg/m3. No entanto existem outras unidades que são largamente usadas tais como: g/cm3; kg/l. Abaixo colocamos uma equivalência que é freqüentemente utilizada para facilitar na hora que precisarmos efetuar conversões de unidades:

1kg/m3 = 10–3 g/cm3

Pressão

Como os fluidos não suportam tensões de cisalhamento (tangenciais) os esforços que são exercidos sobre os fluidos ou os esforços que os fluidos exercem sobre os corpos devem ser medidos através da grandeza física escalar que recebe o nome de pressão e é definida como sendo a razão entre o módulo da força exercida e a área que a força atua. Sendo assim temos:

A

Fp=

Observe que representamos a pressão por p (minúsculo) afim de não confundirmos em algum problema pressão com peso (representado por P (maiúsculo). A unidade de pressão no SI é o N/m2 (Newton por metro quadrado), que recebe o nome especial de Pa (pascal), isto é, 1N/m2 = 1Pa. No entanto existem outras unidades de pressão que são largamente usadas tais como: atm (atmosfera), cmHg (centímetro de mercúrio). Abaixo mostramos a equivalência entre as unidades de pressão mais usadas: facilitar na hora que precisarmos efetuar conversões de unidades:

105 Pa = 1atm = 76cmHg = 760mmHg

Teorema de Stevin

Podemos enunciar o teorema de Stevin da seguinte forma: A DIFERENÇA DE PRESSÃO ENTRE DOIS PONTOS DE UM FLUIDO

HOMOGÊNEO EM EQUILÍBRIO É DADA PELA PRESSÃO HIDROSTÁTICA DA COLUNA DE FLUIDO ENTRE ESTES DOIS PONTOS.

Matematicamente temos:

321cahidrostátipressão

hgρpp AB ⋅⋅+=

g ���� aceleração da gravidade ρ ���� densidade do fluido Se o fluido tem a superfície livre, isto é, o ponto A é a superfície do fluido nesta atua a pressão atmosférica do local, representando a pressão atmosférica por p0 o teorema de Stevin para o cálculo da pressão p num ponto a uma profundidade h da superfície do fluido pe dado por:

hgρpp 0 ⋅⋅+=

h = 0,1 m

A

B

k1

k2

centro

m

k1 = k2 = k

B

O

A

Q

P

corpo

régua

mA

mB

A

B

C

P

2,4m

1,2m

A

B

h

FÍSICA – MECÂNICA

AUTORIA – PROF. MARCELO CORREIA E-mail: [email protected] 23

Teorema de Pascal Podemos enunciar o teorema de Stevin da seguinte forma: UMA VARIAÇÃO DE PRESSÃO PROMOVIDA NUM PONTO QUALQUER DE UM FLUIDO EM EQUILÍBRIO, SE TRANSMITE INTEGRALMENTE PARA

TODOS OS PONTOS DO FLUIDO. A principal aplicação do teorema de pascal é o que chamamos de prensa hidráulica. A prensa hidráulica consiste dois recipientes verticais (comumente cilíndricos) de secções retas distintas interligadas por um tubo, no interior do dispositivo é colocado um fluido que sustenta dois êmbolos móveis. Abaixo mostramos uma figura esquemática da prensa hidráulica e a equação deduzida a partir do teorema de pascal:

2

2

1

1

A

F

A

F=

Teorema de Arquimedes ou Teorema do Empuxo Podemos enunciar o teorema de Stevin da seguinte forma:

TODO CORPO TOTAL OU PARCIALMENTE IMERSO NUM FLUIDO EM EQUILÍBRIO, FICA SUJEITO A AÇÃO DE UMA FORÇA VERTICAL E APONTANDO PARA CIMA QUE RECEBE O NOME DE EMPUXO E TEM MÓDULO IGUAL AO PESO DE FLUIDO DESLOCADO PELO CORPO.

Matematicamente podemos escrever:

odacolsed odiulfPE =

Depois de aplicar a definição de peso e densidade podemos chegar a seguinte expressão equivalente à anterior:

gVρE fdf ⋅⋅=

Onde: ρf ���� densidade do fluido; Vfd ���� volume de fluido deslocado e g ���� aceleração da gravidade

BRINQUEDINHO DE VESTIBULANDO!!!!!

172. (COVEST) Uma mola ideal de comprimento L=65cm está presa no fundo de uma piscina que está sendo cheia. Um cubo de isopor de aresta a=10cm e massa desprezível é preso na extremidade superior da mola. O cubo fica totalmente coberto no instante em que o nível da água atinge a altura H=1,0m em relação ao fundo da piscina. Calcule a constante elástica da mola, em N/m.

173. (COVEST) Um bloco homogêneo e impermeável, de densidade ρρρρ=0,25g/cm3, está em repouso, imerso em um tanque completamente cheio de água e vedado, como mostrado na figura a seguir. Calcule a razão entre os módulos da força que o bloco exerce na tampa superior do tanque e do peso do bloco.

174. (COVEST) Um tubo em U, aberto em ambas as extremidades e de seção reta uniforme, contém uma certa quantidade de água. Adiciona-se 500mL de um líquido imiscível, de densidade ρρρρ=0,8g/cm3, no ramo da esquerda. Qual o peso do êmbolo, em newtons, que deve ser colocado no ramo da direita, para que os níveis de água nos dois ramos sejam iguais? Despreze o atrito do êmbolo com as paredes do tubo.

175. (COVEST) A figura mostra dois recipientes, cujas bases têm áreas que satisfazem à relação A1=3A2. Coloca-se 33 litros de água nestes recipientes, até atingir o nível h. Determine a força exercida pela água sobre a base do recipiente 2, em kgf. Despreze o efeito da pressão atmosférica.

176. (COVEST) É impossível para uma pessoa respirar se a diferença de pressão entre o meio externo e o ar dentro dos pulmões for maior do que 0,05atm. Calcule a profundidade máxima, h, dentro d’água, em cm, na qual um mergulhador pode respirar por meio de um tubo, cuja extremidade superior é mantida fora da água.

177. (COVEST) Duas esferas de mesmo raio e massas MA=0,5 kg e

MB=0,3 kg, estão presas por um fio fino, inextensível e de massa desprezível, conforme mostra a figura. As esferas encontram-se em repouso, imersas em um líquido. Determine o empuxo exercido pelo líquido sobre cada esfera.

178. (UPE) A famosa experiência de Torricelli foi realizada com o mercúrio, porque:

(a) Se fosse feita com a água, que apresenta densidade muito inferior à do mercúrio, a altura seria imperceptível.

(b) Se fosse feita com um líquido mais denso que o mercúrio, o tubo de vidro deveria ter maior comprimento.

(c) O mercúrio é o único metal em estado líquido, na temperatura ambiente. (d) O mercúrio, sendo um metal líquido, é bom condutor de calor. (e) Se fosse feita com a água, com densidade muito menor que a do mercúrio,

o tubo de vidro deveria ter comprimento maior que 10 m.

F1

F2

A1

A2

H

L

a

tampa

água

êmbolo

água

líquido

A1 A2

h

recipiente 1 recipiente 2

MB